Vous êtes sur la page 1sur 78

Foundation Level

Business Law

3b
INSTRUCTIONS TO CANDIDATES Read this page before you look at the questions

FBLW
21 May 2001 Day 1 early afternoon

You are allowed TWO hours to answer this question paper. Answer the ONE question in section A (this has 25 sub-questions). Answer TWO questions ONLY from section B. Write your examination number in the boxes provided on the front of the answer book. Write FBLW on the line marked "Subject" on the front of the answer book. Write your examination number on the special answer sheet for section A which is on page 3 of this question paper booklet. Detach the sheet from the booklet and insert it into your answer book before you hand this in. Do NOT write your name or your student registration number anywhere on your answer book. Tick the appropriate boxes on the front of the answer book to indicate which questions you have answered.

The Chartered Institute of Management Accountants 2001

FOR FREE CIMA, ACCA & CAT RESOURCES VISIT: http://kaka-pakistani.blogspot.com

SECTION A 50 MARKS ANSWER ALL TWENTY-FIVE SUB-QUESTIONS 2 MARKS EACH


Each of the sub-questions numbered from 1.1 to 1.25 inclusive, given below, has only ONE correct answer. Question One 1.1 A B C D Which ONE of the following is the highest court in England? The Court of Appeal. The House of Lords. The High Court. The Crown Court.

1.2

Which part of a case decided by the courts is binding on lower courts dealing with the same material facts? Obiter dicta. The decision of the judge. The ratio decidendi. All the above.

A B C D

1.3 A B C D

In the tort of negligence, damages are payable in respect of reasonably foreseeable losses. losses which are a direct consequence of the breach of duty. all the losses caused by the breach of duty. losses which are within the contemplation of the parties.

1.4

Which of the following statements is/are correct? (i) (ii) An employer is vicariously liable for the torts of employees committed in the course of their employment. An employer is vicariously liable for the torts of independent contractors, if they were committed whilst carrying out work for the employer.

A B C D

(i) only. (ii) only. Both (i) and (ii). Neither (i) nor (ii).

FBLW

May 2001

FOR FREE CIMA, ACCA & CAT RESOURCES VISIT: http://kaka-pakistani.blogspot.com

1.5

Which of the following statements is/are correct? (i) (ii) A contract of employment must be in writing. An employer must provide written particulars of the employment for the employee.

A B C D

(i) only. (ii) only. Both (i) and (ii). Neither (i) nor (ii).

1.6 A B C D

Which ONE of the following statements is incorrect? It is automatically unfair to dismiss an employee for trade union activity. It is automatically unfair to dismiss an employee who becomes pregnant. It is automatically unfair to dismiss an employee who enforces a statutory right. It is automatically unfair to dismiss an employee who refuses to obey a reasonable instruction.

1.7 A B C D

Which ONE of the following is incorrect? An employer is obliged to provide an employee with a reference. An employer must allow Health and Safety representatives reasonable paid time off work to perform their functions. An employer must provide an employee with a safe working environment. Employees who have three months continuous employment are entitled to paid leave.

1.8

Which of the following statements is/are correct? (i) If an agreement is stated to be binding in honour only, the parties have decided that the agreement should not have contractual force. If an agreement is not in writing, the parties are presumed to have intended that it should not be legally enforceable.

(ii)

A B C D

(i) only. (ii) only. Neither (i) nor (ii). Both (i) and (ii).

May 2001

FBLW

FOR FREE CIMA, ACCA & CAT RESOURCES VISIT: http://kaka-pakistani.blogspot.com

1.9 A B C D

A contract entered into as a result of a misrepresentation is enforceable. voidable. void. absolutely valid.

1.10 Which ONE of the following does NOT discharge a contract? A B C D Precise performance of all the contractual obligations. A subsequent event causing the contract to be impossible to perform. Breach of a warranty. The agreement of the parties, contained in a deed, to the effect that the contract should be discharged.

1.11 Which ONE of the following remedies for breach of contract must be awarded by the court if there has been a breach of contract? A B C D An injunction. Damages. Specific performance. Recission.

1.12 Which ONE of the following statements is incorrect? A B C D Payment by the debtor of less than the full amount of the debt will satisfy the whole debt at common law, if paid early at the request of the debtor. Payment by a third party of less than the full amount of the debt will discharge the whole debt at common law. Payment of less than the full amount of the debt will discharge the whole debt at common law, if agreed to by the creditor. Payment of less than the full amount of the debt will discharge the whole debt at common law, if extra consideration is provided by the debtor.

FBLW

May 2001

FOR FREE CIMA, ACCA & CAT RESOURCES VISIT: http://kaka-pakistani.blogspot.com

1.13 Which of the following statements is/are correct? (i) (ii) A contractual term which attempts to exclude liability for damage to property caused by negligence is void unless reasonable. A contractual term which attempts to exclude liability for death or personal injury is void.

A B C D

(i) only. (ii) only. Neither (i) nor (ii). Both (i) and (ii).

1.14 In relation to company law, which ONE of the following statements is correct? A B C D A company limited by shares is fully liable for all its debts. Shareholders are fully liable for the debts of the company. Directors are fully liable for the debts of the company. A company secretary is fully liable for the debts of the company.

1.15 Which ONE of the following statements in relation to partnership law is incorrect? A B C D In England, a partnership has no existence separate from the partners. Each partner can bind the firm in contract if acting in the ordinary course of business. Partners are not liable for debts contracted before they became partners. To be binding, a partnership agreement must be in writing.

1.16 Which of the following statements is/are correct? (i) (ii) A B C D A companys authorised share capital must be stated in its Memorandum of Association. A company may not increase its authorised share capital.

(i) only. (ii) only. Both (i) and (ii). Neither (i) nor (ii).

May 2001

FBLW

FOR FREE CIMA, ACCA & CAT RESOURCES VISIT: http://kaka-pakistani.blogspot.com

1.17 Which ONE of the following statements is correct in relation to companies limited by shares? A B C D All new shares issued must be offered to the existing members first. All new shares issued for cash must be offered to the existing members first in the case of a public company only. All new shares must be offered to the existing members first in the case of a private company. All shares issued for cash must be offered to the existing members first.

1.18 Which ONE of the following statements is incorrect? A B C D A private company limited by shares must have at least one director. A public company limited by shares must have at least two directors. A private company must have authorised share capital of at least 30,000. A public company must have an authorised share capital of at least 50,000.

1.19 In order for a private company to alter its Articles of Association, the shareholders must pass A B C D a special resolution. an ordinary resolution. an extraordinary resolution. an elective resolution.

1.20 In relation to directors, which ONE of the following statements is incorrect? A B C D The board of directors is the agent of the company. The board of directors is the agent of the shareholders. The board may appoint a person to be the managing director. Proceedings at board meetings are regulated by the companys Articles of Association.

FBLW

May 2001

FOR FREE CIMA, ACCA & CAT RESOURCES VISIT: http://kaka-pakistani.blogspot.com

1.21 Which ONE of the following statements is/are correct? (i) (ii) In general, it is illegal for a company to provide financial assistance for the purchase of its own shares. A private company may provide financial assistance for the purchase of its own shares as long as it follows the correct procedure.

A B C D

(i) only. (ii) only. Both (i) and (ii). Neither (i) nor (ii).

1.22 Which ONE of the following resolutions is required to enable a company to borrow money? A B C D A special resolution. An ordinary resolution. An extraordinary resolution. A board resolution.

1.23 Which ONE of the following statements is incorrect? A B C D The Articles of Association form a contract between the shareholders and the board. The Articles of Association form a contract between the shareholders and the company. The Articles of Association form a contract between each shareholder and the other shareholders. The Articles of Association are only contractual in respect of ordinary membership rights.

1.24 Which ONE of the following statements is correct? A B C D A floating charge has priority over a fixed charge. The preferential creditors take priority over fixed charge holders. A fixed charge has priority over a floating charge. Unsecured creditors take priority over floating charge holders.

May 2001

FBLW

FOR FREE CIMA, ACCA & CAT RESOURCES VISIT: http://kaka-pakistani.blogspot.com

1.25 In relation to elective resolutions, which ONE of the following statements is incorrect? A B C D The resolution must be passed by all the members. The resolution applies only to private companies. The resolution may be used to dispense with the need to lay accounts before the general meeting. The resolution must be agreed to by all the members present and voting.

(Total = 50 marks)

FBLW

May 2001

FOR FREE CIMA, ACCA & CAT RESOURCES VISIT: http://kaka-pakistani.blogspot.com

SECTION B 50 MARKS ANSWER TWO QUESTIONS ONLY


Question Two

(a) Identify and explain the sources of English Law. (15 marks) (b) Bee plc owned 5% of the shares in Cee plc and was considering making a take-over bid for
the shares in Cee plc. In December 2000, Bee plc wrote to Cee plc's auditors, A & Co, asking if their audit report could be relied upon as representing an accurate review of the financial position of Cee plc. Tom, the senior partner of A & Co, replied in writing that Bee plc could indeed rely on the accuracy of their report. As a result, Bee plc submitted a takeover bid and gained control of the entire share capital of Cee plc. Bee plc has now discovered that Cee plc's assets are worth far less than stated by A & Co in their audit report. Bee plc considers this to have been caused by A & Co carelessly overvaluing Cee plc's assets. This resulted in Bee plc paying more for the shares than it would have done had the truth been known.

Required:
As the Finance Director of Bee plc, write an internal memorandum to the board explaining whether Bee plc appears to be entitled to receive any compensation from A & Co.

(10 marks) (Total = 25 marks)

Question Three

(a) By reference to examples, explain:


(i)
the remedies available to the innocent party in the event of a breach of a condition or a warranty;

(12 marks)
(ii) how the law determines whether a contractual term is a condition or a
warranty.

(8 marks) (b) Charlotte is a self-employed Business Consultant. She contracted with Zed plc to carry out
a thorough review of the company's operations. The review was to last 8 weeks and to commence on 5 February 2001. The contract included the following terms: Clause 1: "It shall be a term of this contract that Charlotte will attend the board meeting of Zed plc to be held on 5 February 2001." Clause 2: "A breach of Clause 1 of this agreement shall entitle Zed plc to terminate the contract." Charlotte was ill on 5 February 2001 and her secretary sent a fax to Zed plc apologising for the fact that she would not be able to start until 9 February 2001. Charlotte has now been advised that Zed plc has terminated her contract.

Required:
Explain whether Charlotte is entitled to any compensation in respect of Zed plc's actions.

(5 marks) (Total = 25 marks)


FBLW 9 M2001

FOR FREE CIMA, ACCA & CAT RESOURCES VISIT: http://kaka-pakistani.blogspot.com

Question Four

(a) In relation to companies limited by shares, explain:


(i)
how directors may be appointed and dismissed;

(8 marks)
(ii) the duties owed by directors to shareholders.

(8 marks) (b) Asif and Ben are the only directors of AB Ltd, each holding 20% of the shares. The
remaining shares are held by Claire, Debbie and Edward, who also each own 20% of the shares. AB Ltd recently entered into a contract with XY Ltd for the construction of new office premises. The building work carried out by XY Ltd proved unsatisfactory, and a considerable amount had to be spent by AB Ltd on remedying the defects. At a board meeting, Asif and Ben decided that the company should not sue XY Ltd, as it would be likely to take up a considerable amount of time and money, and, in any event, it seemed unlikely that XY Ltd would be able to pay. Claire, Debbie and Edward strongly object to the decision and have written a letter to the board insisting that the company should sue XY Ltd.

Required:
As the Company Secretary of AB Ltd, write an internal memorandum to the board explaining:

(i)

whether the shareholders can force the directors to cause AB Ltd to sue XY Ltd;

(6 marks)
(ii) whether the shareholders themselves can sue XY Ltd.

(3 marks) (Total = 25 marks)

End of paper

M2001

10

FBLW

FOR FREE CIMA, ACCA & CAT RESOURCES VISIT: http://kaka-pakistani.blogspot.com

Foundation Level

Business Law

3b
INSTRUCTIONS TO CANDIDATES Read this page before you look at the questions

FBLW
19 November 2001 Day 1 early afternoon

You are allowed TWO hours to answer this question paper. Answer the ONE question in section A (this has 25 sub-questions, and is on pages 2 7). Answer TWO questions ONLY from section B (these questions are on pages 8 and 9). Write your examination number in the boxes provided on the front of the answer book. Write FBLW on the line marked "Subject" on the front of the answer book. Write your examination number on the special answer sheet for section A. Detach the sheet from the booklet and insert it into your answer book before you hand this in. Do NOT write your name or your student registration number anywhere on your answer book. Tick the appropriate boxes on the front of the answer book to indicate which questions you have answered.

The Chartered Institute of Management Accountants 2001

FOR FREE CIMA, ACCA & CAT RESOURCES VISIT: http://kaka-pakistani.blogspot.com

SECTION A 50 MARKS ANSWER ALL TWENTY-FIVE SUB-QUESTIONS 2 MARKS EACH


Each of the sub-questions numbered from 1.1 to 1.25 inclusive, given below, has only ONE correct answer.

Question One 1.1 Which of the following statements is correct? (i) In the event of a conflict between equity and the common law, the common law prevails. An Act of Parliament can overrule any common law or equitable rule. B (ii) only. C Neither (i) nor (ii). D Both (i) and (ii).

(ii) A

(i) only.

1.2 A B C D

All criminal cases commence in the County Court. the Crown Court. the Court of Appeal. the Magistrates Court.

1.3 A B C D

In the tort of negligence, if a claimant is partly responsible for his/her own injuries, then no compensation can be recovered from the defendant. the defendant is fully liable if he/she was mainly responsible for the injuries. if the defendant was negligent, he/she remains fully liable for all the injuries caused. the compensation will be reduced to take account of the claimants share of the responsibility.

1.4

An employer must provide an employee with a written statement of particulars of the employment within one month of the employment commencing. as soon after the commencement of employment as possible. within two months of the employment commencing. within a reasonable time of the employment commencing.

A B C D

FBLW

November 2001

FOR FREE CIMA, ACCA & CAT RESOURCES VISIT: http://kaka-pakistani.blogspot.com

1.5 A B C D

Which ONE of the following statements is correct? An employer is obliged to provide a careful and honest reference. An employer is obliged to provide a safe system of work. An employer is obliged to provide employees with smoking facilities during authorised breaks at work. An employer with fewer than 20 employees is obliged to provide an itemised written pay statement.

1.6

The vast majority of contracts are "simple". What is the meaning of the word "simple" in this context? The terms of the contract are set out in writing. The contract does not need to be in any particular form to be binding. The contract contains fewer than ten provisions. The contract is not supported by consideration.

A B C D

1.7

A Ltd placed the following advertisement in a local newspaper: "We are able to offer for sale a number of portable colour television sets at the specially reduced price of 590. Order now while stocks last." The advertisement contained a mistake in that the television sets should have been priced at 5900. B Ltd immediately placed an order for 100 television sets. Which ONE of the following statements is correct?

A B C D

B Ltd has accepted an offer and is contractually entitled to the 100 television sets. A Ltd can refuse to supply B Ltd as the advertisement is not an offer, but an invitation to treat. A Ltd can only refuse to sell the television sets to B Ltd if it has sold all its stock. As B Ltd has not yet paid for the television sets, the company has no contractual right to them.

1.8 A B C D

Which ONE of the following statements is correct? If a person signs a contract, he/she is bound by all its terms. A contract which has not been signed is not binding on any of the parties. A person who signs a contract is deemed to have read it. A person who has not read a contract cannot be bound by it.

November 2001

FBLW

FOR FREE CIMA, ACCA & CAT RESOURCES VISIT: http://kaka-pakistani.blogspot.com

1.9 A B C D

Which ONE of the following statements is correct? If the creditor agrees to accept less than the full amount due, the debt is discharged at common law. At common law, a creditor who has agreed to accept less than the full amount due, may go back on his word and recover the balance. Payment of less than the full amount due by a third party cannot discharge the whole debt. Payment of less than the amount due cannot discharge the whole debt, even if made early at the request of the creditor.

1.10 Which ONE of the following is not an equitable remedy? A B C D Damages. Specific performance. Rescission. Injunction.

1.11 Which of the following statements is correct? (i) As a general rule, a contract will only be discharged if all its terms have been precisely performed. If a contract becomes impossible to perform through no fault of either contracting party, the contract is frustrated and unenforceable, unless its terms provide for the frustrating event. B (ii) only. C Neither (i) nor (ii). D Both (i) and (ii).

(ii)

(i) only.

1.12 In breach of contract, C Ltd refused to sell a motor car to D Ltd at the agreed price of 10,000. If the type of motor car is readily available on the market at a price of 9,000, which ONE of the following is correct? A B C D D Ltd is entitled to an order of specific performance, forcing C Ltd to carry out its contract. D Ltd is entitled to damages of 1,000. D Ltd is entitled to nominal damages only. D Ltd is not entitled to damages.

FBLW

November 2001

FOR FREE CIMA, ACCA & CAT RESOURCES VISIT: http://kaka-pakistani.blogspot.com

1.13 In relation to E Ltd, a company limited by shares, which ONE of the following statements is correct? A B C D The liability of the company and its shareholders is limited, but the directors are fully liable for the companys debts. The liability of the company and its directors is limited, but the shareholders are fully liable for the companys debts. The liability of the company, its directors and shareholders is limited. The liability of the directors and shareholders is limited, but the company is fully liable for its own debts.

1.14 Adam, Ben and Carol have carried on business together in partnership since July 2001. In September 2001, they decided to enter into a formal partnership agreement. The partners agreed the terms of the agreement in October 2001 and signed the completed agreement in November 2001. According to the law, when did the partnership commence? A July 2001. B September 2001. C October 2001. D November 2001.

1.15 Which of the following statements is correct? (i) It is not possible to register a company limited by shares with the same name as a company already on the register. Once on the register, a company limited by shares cannot change its registered office. B (ii) only. C Both (i) and (ii). D Neither (i) nor (ii).

(ii)

(i) only.

1.16 The authorised share capital of Wye Ltd is 250,000 divided into 250,000 ordinary 1 shares. The asset value of each share is 2. Angela and Brian are the only shareholders. Each has taken 50,000 shares and each has, so far, paid 10,000. Which ONE of the following statements is correct? A B C D The issued share capital of Wye Ltd is 250,000, and the paid up capital is 20,000. The issued share capital of Wye Ltd is 100,000, and the paid up capital is 20,000. The issued share capital of Wye Ltd is 250,000, and the paid up capital is 100,000. The issued share capital of Wye Ltd is 20,000, and the paid up capital is 20,000.

November 2001

FBLW

FOR FREE CIMA, ACCA & CAT RESOURCES VISIT: http://kaka-pakistani.blogspot.com

1.17 Which of the following statements is correct? (i) The Articles of Association of a company limited by shares contain the internal regulations of the company. The Articles of Association form a contract between the shareholders and the company. B (ii) only. C Both (i) and (ii). D Neither (i) nor (ii).

(ii)

(i) only.

1.18 Which of the following resolutions may be used to increase a companys authorised capital? (i) (ii) A Ordinary resolution. Written resolution. B (ii) only. C Both (i) and (ii). D Neither (i) nor (ii).

(i) only.

1.19 XYZ plc has issued shares on terms that they will be bought back by the company 12 months after the date of issue. What are these shares called? A B C D Ordinary shares. Bonus shares. Preference shares. Redeemable shares.

1.20 Which ONE of the following statements is INCORRECT in relation to elective resolutions? A B C D An elective resolution may only be passed by private companies. An elective resolution may be used to reduce the majority of votes necessary to authorise short notice to 90%. An elective resolution may be used to authorise directors to issue shares for longer than the usual five years. An elective resolution may be used to dispense with the requirement to re-elect directors annually.

1.21 Which ONE of the following is INCORRECT? A B C D A public company must have at least two directors. A private company must have at least one director. Both public and private companies must have a qualified company secretary. A public company must have at least two shareholders.

FBLW

November 2001

FOR FREE CIMA, ACCA & CAT RESOURCES VISIT: http://kaka-pakistani.blogspot.com

1.22 Immediately before XY Ltd was placed in insolvent liquidation, Alex, the companys sole director, arranged for the company to make an early repayment of an unsecured loan of 15,000 which he had provided to the company. Which ONE of the following is correct? A B C D The repayment may amount to a "preference", and Alex may be required to hand back the 15,000 to XY Ltd. Alex may be fined. The repayment of the loan is valid so long as Alex was acting in good faith. XY Ltd and Alex may be guilty of fraud.

1.23 The court has decided that Jill, a director of Jay Ltd, has been wrongfully trading, in that she continued to carry on business at a time when she should have known that insolvency was inevitable. What are the possible consequences for Jill? A B C D Jill may be fined. Jill may be imprisoned. Jill may be required to contribute to the assets of Jay Ltd. Jill may be required to sell her shares in Jay Ltd.

1.24 Jack has acted in breach of his fiduciary duty as a director of JK Ltd. If the breach does not amount to fraud on the minority, which ONE of the following is correct? A B C D The breach cannot be ratified by the shareholders. The breach may be ratified by a written or ordinary resolution. The breach may be ratified by a provision in the companys Memorandum of Association. The breach may be ratified by a resolution of the board of directors.

1.25 Zed plc holds its board meetings on the fifteenth day of each month. At the meeting on 15 June 2001, the board discussed a potential contract with RST Ltd. On 1 July 2001, Lucy, a director of Zed plc, bought shares in RST Ltd. On 25 July 2001, Zed plc contracted with RST Ltd. When should Lucy have declared her interest to the board of Zed plc? A 15 June 2001. B 1 July 2001. C 15 July 2001. D 15 August 2001.

(Total = 50 marks)

November 2001

FBLW

FOR FREE CIMA, ACCA & CAT RESOURCES VISIT: http://kaka-pakistani.blogspot.com

SECTION B 50 MARKS ANSWER TWO QUESTIONS ONLY


Question Two

(a) Explain how the law determines whether negotiating parties have reached agreement. (15 marks) (b) In September 2001, Exe Ltd ("Exe") telephoned Seller Ltd ("Seller") and ordered 5,000
computer speaker systems at a price of 150,000. It was agreed that the goods would be delivered within five days and that payment should be made within 28 days of the receipt of the goods. Following the telephone conversation, Seller sent Exe a document described as a "sales contract" which set out Seller's terms and conditions. The sales contract contained a tear-off "acknowledgement" section which Exe was asked to sign and return to Seller. One week later, Exe signed the acknowledgement and returned it to Seller together with a letter which stated that the contract was to be governed by Exe's terms and conditions, a copy of which was enclosed. The speaker systems were delivered and Exe paid the 150,000 by the due date. Exe has now received a demand from Seller for 3,000 "transport costs", and has been referred to one of Seller's terms which provides, in small print: "If the purchaser does not arrange for the collection of the goods purchased, the seller will arrange for them to be transported subject to reimbursement by the purchaser who is fully liable for the cost". Exe is refusing to pay, and has referred Seller to its own terms and conditions which include the statement: "Any price which Exe Ltd agrees to pay shall be deemed to include all transport costs".

Required:
As the Company Secretary of Seller Ltd, write a memorandum to your Sales Director, explaining whether the company is entitled to recover the 3,000 transport costs from Exe Ltd.

(10 marks) (Total = 25 marks)

Question Three

(a) Explain how the law determines whether an individual is an employee or an independent
contractor.

(13 marks) (b) Tom was employed as a mechanic by Cee plc. In October 2001, he had an argument with
Keith, the workshop manager, who lost his temper and told Tom to collect his possessions and leave, as he was dismissed with immediate effect.

Required:
Prepare a memorandum, in your role as Company Secretary, addressed to the Senior Human Resources Officer of Cee plc, explaining whether Tom has any remedies against Cee plc.

(12 marks) (Total = 25 marks)

November 2001

FBLW

FOR FREE CIMA, ACCA & CAT RESOURCES VISIT: http://kaka-pakistani.blogspot.com

Question Four Danny, Edward and Fozia each hold one third of the issued share capital of DEF Ltd. Danny's service contract states that he is to be the company's sole director until May 2003. Recently, DEF Ltd contracted to borrow a substantial sum of money from Zed Bank plc in order to re-equip the company's production lines, which Danny considered to be inefficient. The bank advised DEF Ltd that the loan would be subject to the company providing security in the form of a fixed charge over the company's factory and a floating charge over the company's stock. Edward and Fozia have objected to Danny's decision to obtain the loan, as they believe that the existing production lines are capable of lasting a further two years. They also believe that Danny should have obtained shareholder approval before committing the company to the loan, and have advised him that they are considering removing him as a director.

Required:
(a) Explain the differences between fixed and floating charges. (8 marks) (b) Identify the steps which may be taken by Zed Bank plc in the event of the company
defaulting on the loan.

(6 marks) (c) Explain whether Edward and Fozia are correct so far as company law is concerned, in
believing that shareholder approval was needed before DEF Ltd could obtain the loan, and whether they can arrange for the company to withdraw from the loan.

(4 marks) (d) Explain whether Edward and Fozia have the ability to remove Danny as a director. (4 marks) (e) Explain whether Edward and Fozia have the ability to change the Articles of Association so
that in future all decisions must be approved by the shareholders.

(3 marks) (Total = 25 marks)

End of paper

FBLW

November 2001

FOR FREE CIMA, ACCA & CAT RESOURCES VISIT: http://kaka-pakistani.blogspot.com

Examination Question and Answer Book


Write here your full examination number Centre Code: Hall Code: Desk Number:

Foundation Level

Business Law

3b
INSTRUCTIONS TO CANDIDATES
Read this page before you look at the questions

FBLW
20 May 2002 Day 1 early afternoon

THIS QUESTION PAPER BOOKLET IS ALSO YOUR ANSWER BOOKLET. Sufficient space has been provided for you to write your answers and also for workings where questions require them. For section B questions, you must write your answers in the shaded space provided. An additional blank page (14) is included towards the back of this booklet if you require more space for notes or workings. Please note that you will NOT receive marks for your notes or workings. Do NOT remove any sheets from this booklet: cross through neatly any work that is not to be marked. Avoid the use of correction fluid. You are allowed two hours to answer this question paper. All questions are compulsory. Answer the ONE question in section A (this has 25 sub-questions and is on pages 2-9) Answer the THREE questions in section B (these are on pages 10-13) You are advised to spend 10 minutes reading through the paper before starting to answer the questions. You should spend no more than 55 minutes on answering the ONE question in section A, which has 25 sub-questions. You should spend no more than 55 minutes on answering the THREE questions in section B. Hand this entire booklet to the invigilators at the end of the examination. You are NOT permitted to leave the examination hall with this booklet. Do NOT write your name or your student registration number anywhere on this booklet. TURN OVER

For office use only


Marks awarded (First marker) for each question Marks awarded (Second marker) for each question
The Chartered Institute of Management Accountants 2002

Total

One

Two

Three

Four

FOR FREE CIMA, ACCA & CAT RESOURCES VISIT: http://kaka-pakistani.blogspot.com

SECTION A 50 MARKS ANSWER ALL TWENTY-FIVE SUB-QUESTIONS 2 MARKS EACH


Each of the sub-questions numbered from 1.1 to 1.25 inclusive, given below, has only ONE correct answer. REQUIRED: Place a circle O around the letter A, B, C or D that gives the correct answer to each sub-question. If you wish to change your mind about an answer, block out your first answer completely and then circle another letter. You will NOT receive marks if more than one letter is circled. Please note that you will NOT receive marks for any workings to these sub-questions.

Question One 1.1 A B C D Which ONE of the following statements is correct? The aim of the criminal law is to regulate behaviour within society by the threat of punishment. The aim of the criminal law is to punish offenders. The aim of the criminal law is to provide a means whereby injured persons may obtain compensation. The aim of the criminal law is to ensure that the will of the majority is imposed upon the minority.

1.2 A B C D

Which ONE of the following is correct? The House of Lords is obliged to apply English Law, even if it contradicts European Law. The House of Lords must apply European Law even if it contradicts English Law. If European and English Law conflict, the House of Lords cannot apply either Law. The House of Lords must apply English Law unless it obtains government permission to apply European Law.

1.3 A B C D

Which ONE of the following is correct? Professional advisers cannot be liable in respect of negligent advice in the tort of negligence, but may be liable for breach of contract. Professional advisers cannot be liable for breach of contract in respect of negligent advice but may be liable in the tort of negligence. Professional advisers may be liable in respect of negligent advice in either contract or tort. Professional advisers cannot be liable in respect of negligent advice in either contract or tort.

For office use only


Marks awarded (First marker) for each sub-question Marks awarded (Second marker) for each sub-question
FBLW 2

Total

1.1

1.2

1.3

May 2002

FOR FREE CIMA, ACCA & CAT RESOURCES VISIT: http://kaka-pakistani.blogspot.com

1.4

What is the legal effect of the following statement in a newspaper?

"For sale. Computer, monitor and laser printer. Good condition. 500."
A B C D The statement is an offer for sale. The statement is a mere puff or boast. The statement has no legal effect. The statement is an invitation to treat.

1.5

On 1 September, Seller Ltd wrote to Buyer Ltd offering to sell a machine at a price of 10,000, and stating that Buyer Ltd must accept by 10 September. On 3 September, Buyer Ltd wrote to Seller Ltd and stated "I accept. Will you accept payment over three months?" On 5 September, Seller Ltd sold the machine to New Ltd, and on 6 September received a second letter from Buyer Ltd accepting the offer and offering to make immediate payment. Which ONE of the following is correct?

There is no contract between Seller Ltd and Buyer Ltd because the offer was withdrawn on 5 September when the machine was sold to New Ltd. There is no contract between Seller Ltd and Buyer Ltd because Buyer Ltds letter of 3 September amounted to a counter-offer which destroyed Seller Ltds original offer. Seller Ltd and Buyer Ltd contracted on 3 September. Seller Ltd and Buyer Ltd contracted on 6 September.

C D

1.6

Which of the following examples of performance amounts to good consideration? (i) (ii) (iii) The performance of an existing duty under the general law. The performance of an existing contract in return for a promise by a third party. The performance of an act, followed by a promise to pay for that act.

A B C D

(i) only. (ii) only. (i) and (ii) only. (iii) only.

TURN OVER

For office use only


Marks awarded (First marker) for each sub-question Marks awarded (Second marker) for each sub-question
May 2002 3

Total

1.4

1.5

1.6

FBLW

FOR FREE CIMA, ACCA & CAT RESOURCES VISIT: http://kaka-pakistani.blogspot.com

1.7 A B C D

Which ONE of the following is INCORRECT? A term may be implied into a contract by statute. A term may be implied into a contract by a court on the ground that the term is customary in the parties' trade. A term may be implied into a contract by a court on the ground that it would make the contract more equitable. A term may be implied into a contract by a court on the ground of business efficacy.

1.8 A B C D

Which ONE of the following is INCORRECT? A condition is a term which the parties intended to be of fundamental importance. A warranty is a term which the parties did not intend to be of fundamental importance. If a condition is breached, then the contract must be terminated. If a warranty is breached, then the innocent party cannot terminate the contract.

1.9 A B

Which ONE of the following contracts might be specifically enforceable? Alan has contracted to sell his house to Bob but has changed his mind and no longer wishes to sell it. Chris has contracted to buy a new Ford motor car but the garage is now refusing to honour the contract. Diane has contracted to purchase a number of tins of fruit for her business but the seller has now stated that he no longer wishes to proceed with the contract. Eduardo has contracted to sing at a concert organised by Fernando, but Eduardo has withdrawn as he has received a more lucrative offer from Giovanni.

1.10 In the event of a breach of contract, what is the purpose of damages? (i) (ii) (iii) A B C D To punish the contract breaker. To compensate the innocent party. To put the innocent party in the same position as if the contract had been carried out correctly.

(i) only. (ii) and (iii) only. (iii) only. (i), (ii) and (iii).

For office use only Marks awarded (First marker) for each sub-question Marks awarded (Second marker) for each sub-question
FBLW 4

Total

1.7

1.8

1.9

1.10

May 2002

FOR FREE CIMA, ACCA & CAT RESOURCES VISIT: http://kaka-pakistani.blogspot.com

1.11 Which of the following statements suggests that John is an independent contractor in relation to the work he carries out for Zed Ltd? (i) (ii) (iii) A B C D He is required to provide his own tools. He is required to carry out his work personally and is not free to send a substitute. He is paid in full without any deduction of income tax.

(i) and (ii) only. (ii) and (iii) only. (i) and (iii) only. (i), (ii) and (iii).

1.12 Which ONE of the following is normally implied into a contract of employment? A B C D A duty to provide a reference. A duty to provide work. A duty to pay wages. An employees duty to disclose his own misconduct.

1.13 A business has been registered under the name "The Mark Jones Partnership Co Ltd". What type of business organisation must this be? A B C D A partnership. A private limited company. A public limited company. Any of the above as this is a business name.

1.14 Which ONE of the following statements is INCORRECT in relation to a public company limited by shares? A B C D The company must have at least one director. The company must have at least two shareholders. The company must have issued at least 50,000 of shares. The company must state in its Memorandum of Association that it is a public limited company.

TURN OVER For office use only Marks awarded (First marker) for each sub-question Marks awarded (Second marker) for each sub-question
May 2002 5

Total

1.11

1.12

1.13

1.14

FBLW

FOR FREE CIMA, ACCA & CAT RESOURCES VISIT: http://kaka-pakistani.blogspot.com

1.15 Which of the following is correct? (i) (ii) (iii) Purchasing a shelf company enables business to commence more quickly. It is generally cheaper to purchase a shelf company than to arrange for a solicitor or accountant to register a new company. Incorporating a company by registration enables the companys documents to be drafted to the particular needs of the incorporators.

A B C D

(i) and (ii) only. (ii) and (iii) only. (i) and (iii) only. (i), (ii) and (iii).

1.16 Which ONE of the following is INCORRECT? A B C D If a companys object is to carry on business as a general commercial company, the company may carry on any trade or business whatsoever. If a company acts outside its objects clause, it has acted ultra vires and the transaction is void. A company may ratify an ultra vires act by passing a special resolution. Any shareholder may apply for an injunction to prevent the directors from taking the company into an ultra vires transaction.

1.17 Which of the following statements is correct? (i) (ii) (iii) The Memorandum of Association of a private company limited by shares may be altered by special resolution. The Memorandum of Association of a private company limited by shares may be altered by a written resolution. The Memorandum of Association of a private company limited by shares may be altered by an ordinary resolution.

A B C D

(i) only. (ii) only. (i) and (ii) only. (ii) and (iii) only.

For office use only Marks awarded (First marker) for each sub-question Marks awarded (Second marker) for each sub-question
FBLW 6

Total

1.15

1.16

1.17

May 2002

FOR FREE CIMA, ACCA & CAT RESOURCES VISIT: http://kaka-pakistani.blogspot.com

1.18 Bee Ltd has an issued share capital of 1,000 ordinary shares of 1 each. Some of the shareholders would like to pass an elective resolution in order to dispense with the need to re-appoint the auditor annually. What is the minimum number of votes which must be cast in order to pass the resolution? A 500. B 501. C 750. D 1,000.

1.19 Which ONE of the following is correct? A B C D A company intending to issue new shares for cash must first offer them to the directors. A company may dispense with the requirement to first offer new shares to the directors by passing a special resolution. A company issuing new shares for cash must first offer them to the existing members. A company issuing new shares for a non-cash consideration must first offer them to the existing members.

1.20 Which ONE of the following is INCORRECT in relation to an increase in a companys authorised share capital? A B C D The board must resolve to increase the authorised capital. The shareholders must pass a resolution to increase the authorised capital. The authorised capital clause is contained in a company's Memorandum of Association. The increase in authorised capital must be confirmed by the court.

1.21 Which ONE of the following is correct? A The shareholders may dismiss a director irrespective of anything in the companys Articles of Association if they pass an ordinary resolution to do so, of which special notice has been given to the company. The shareholders may dismiss a director irrespective of anything in the companys Articles of Association if they pass an extraordinary resolution to do so, of which special notice has been given to the company. The shareholders may dismiss a director irrespective of anything in the companys Articles of Association if they pass a written resolution to do so, of which special notice has been given to the company. The shareholders may dismiss a director irrespective of anything in the companys Articles of Association if they pass an elective resolution to do so, of which special notice has been given to the company.

TURN OVER For office use only Marks awarded (First marker) for each sub-question Marks awarded (Second marker) for each sub-question
May 2002 7

Total

1.18

1.19

1.20

1.21

FBLW

FOR FREE CIMA, ACCA & CAT RESOURCES VISIT: http://kaka-pakistani.blogspot.com

1.22 Which of the following is correct? (i) (ii) (iii) A B C D Directors may vote themselves such salary payments as they think fit, irrespective of anything in the companys Articles of Association. Directors are only entitled to be paid for their services if the constitution of the company so provides. Directors must be paid a salary.

(i) only. (ii) only. (i) and (ii) only. (iii) only.

1.23 Which of the following is correct? (i) (ii) (iii) Individual shareholders cannot interfere with the management of the company unless authorised by the companys Articles of Association. The majority shareholders can interfere with the management of the company unless prevented by the companys Articles of Association. Irrespective of anything in the companys Articles of Association, the directors must act in accordance with the directions issued by the shareholders in the form of special resolutions.

A B C D

(i) only. (ii) only. (i) and (ii) only. (iii) only.

1.24 Which of the following can enforce the fiduciary duties owed by a director? (i) (ii) (iii) The majority shareholders. The company. Individual directors.

A B C D

(i) and (ii) only. (ii) only. (ii) and (iii) only. (iii) only.

For office use only Marks awarded (First marker) for each sub-question Marks awarded (Second marker) for each sub-question
FBLW 8

Total

1.22

1.23

1.24

May 2002

FOR FREE CIMA, ACCA & CAT RESOURCES VISIT: http://kaka-pakistani.blogspot.com

1.25 Which ONE of the following correctly describes the circumstances in which directors may be held liable to contribute to the assets of insolvent companies in respect of wrongful trading? A B C D Where the directors have the intention of defrauding creditors. Whenever a companys liabilities exceed its assets. Whenever a company becomes insolvent. Where directors knew or ought to have known that insolvency was inevitable.

(Total = 50 Marks)

End of Section A

Section B begins on page 10

TURN OVER For office use only Marks awarded (First marker) for each sub-question Marks awarded (Second marker) for each sub-question
May 2002 9

Total

1.25

FBLW

FOR FREE CIMA, ACCA & CAT RESOURCES VISIT: http://kaka-pakistani.blogspot.com

SECTION B 50 MARKS ANSWER ALL THREE QUESTIONS


IMPORTANT MARKS ARE AWARDED FOR CORRECTLY COMPLETING THE SHADED BOXES WITH THE CORRECT ANSWER WHERE A MARK IS INDICATED IN THE RIGHT-HAND COLUMN. DO NOT WRITE IN THE MARGINS NOR IN THE COLUMNS FOR USE BY MARKERS.

Question Two George Thompson has carried on business for a number of years as a self-employed retailer of office furniture. He has now decided to incorporate his business, and to register GT Limited to acquire the business. Do not write in these columns below

Required (a)
Complete this sentence: "In order to register a private limited company, George will need to submit certain documents to the who will issue a certificate of incorporation if everything is in order." (maximum of three words in the gap) Complete these sentences: "One of the documents which George will need to submit contains the company's name, the situation of the company's registered office, a statement that the liability of the members is limited, and the authorised share capital and its division into shares. This document is called the and also contains the which sets out the business(es) which the company is authorised to carry on." (maximum of three words in each gap) In no more than 30 words (in the shaded box below), explain how GT Limited's proposed Articles of Association may be affected by the provisions of Table A.

Marks available

For use by the second marker

For use by the first marker

(b)

(c)

4 Sub-total: 10

Parts (d) and (e) of Question Two are on page 11

FBLW

10

May 2002

FOR FREE CIMA, ACCA & CAT RESOURCES VISIT: http://kaka-pakistani.blogspot.com

Question Two continued Do not write in these columns below


For use by the second marker For use by the first marker

Required (d)
Complete this sentence: "If, at a later date, George wishes to increase the authorised share capital of the company, he may do so by passing resolution." (maximum of two words in the gap) In no more than 30 words (in the shaded area below), explain the liability of both the company and George in the event of the business becoming insolvent.

Marks available

(e)

Sub-total: 6

Total for Question Two = 16 Marks

TURN OVER
May 2002 11 FBLW

FOR FREE CIMA, ACCA & CAT RESOURCES VISIT: http://kaka-pakistani.blogspot.com

Question Three During the sale of a motor car by S (Motor Dealers) Ltd to Anne, the company told her that the car had travelled only "5,000 miles since the installation of a new engine and gearbox". After purchasing the car, Anne paid 700 to have the car repainted, but later discovered that it had travelled more like 50,000 miles since the installation. She attempted to return the car and obtain a refund, but S (Motor Dealers) Ltd refused because it had honestly believed its statement to be correct. As a result, Anne is considering taking legal action against the company for misrepresentation.

Please do not write in these columns below


For use by the second marker For use by the first marker

Required: (a)
In no more than 30 words (in the shaded area below), define misrepresentation.

Marks available

(b)

State (in the shaded area below), the three types of misrepresentation.
2 2 2

(c)

In no more than 30 words (in the shaded box below), identify the type(s) of misrepresentation S (Motor Dealers) Ltd has made and the remedy available to Anne.

(d)

In no more than 30 words (in the shaded area below), explain whether the statement by S (Motor Dealers) Ltd to Anne is also a term of the contract.

4 Total: 18

Total for Question Three = 18 Marks

FBLW

12

May 2002

FOR FREE CIMA, ACCA & CAT RESOURCES VISIT: http://kaka-pakistani.blogspot.com

Question Four Greg, Harry and Ian are the sole shareholders of GHI Ltd. The company has an authorised share capital of 100,000 divided into 100,000 ordinary 1 shares. Greg is the sole director of the company and Harry is the company secretary. At present, the issued share capital is held as follows: Greg: Harry: Ian: Total: 51,000 shares 29,000 shares 20,000 shares 100,000 shares

The company, which has been valued at 300,000, has decided on the following transactions: To raise 60,000 in cash by issuing shares to Fiona. To borrow 40,000 from AB Bank plc.

Required:

Do not write in these columns below

(a)

(b)

Complete the following sentences: "In order to facilitate the issue of the new shares, the authorised capital must be increased by at least ordinary 1 shares." (one figure in the shaded area) "In order to issue the shares, Greg must be authorised by the shareholders or by the (maximum of three words in the shaded area) "Fiona will become a member of the company when she has been entered in the . She must also be sent a within two months

Marks Available

For use by the second marker

For use by the first marker

(c)

(d)

of allotment." (maximum of three words in each of the shaded areas) In no more than 30 words (in the shaded area below), explain how Greg's voting power within GHI Ltd will be affected by the issue of shares to Fiona.

(e)

"If AB Bank plc requires security in the form of a debenture containing a charge, then the company will not be able to deal freely with the assets charged in the ordinary course of business. However, GHI Ltd will be able to deal freely with assets which are secured by a charge." (one word in each shaded area)
Total: 16

Total for Question Four = 16 Marks

End of Question Paper


TURN OVER FOR ADDITIONAL SPACE FOR WORKINGS AND NOTES
May 2002 13 FBLW

FOR FREE CIMA, ACCA & CAT RESOURCES VISIT: http://kaka-pakistani.blogspot.com

You may use this sheet for notes and workings (no marks are awarded for workings)

FBLW

14

May 2002

FOR FREE CIMA, ACCA & CAT RESOURCES VISIT: http://kaka-pakistani.blogspot.com

DO NOT WRITE ON THIS SHEET

May 2002

15

FBLW

FOR FREE CIMA, ACCA & CAT RESOURCES VISIT: http://kaka-pakistani.blogspot.com

3b

FBLW

Business Law

Day 1 early afternoon

FBLW

16

May 2002

FOR FREE CIMA, ACCA & CAT RESOURCES VISIT: http://kaka-pakistani.blogspot.com

Examination Question and Answer Book


Write here your full examination number Centre Code: Hall Code: Desk Number:

Foundation Level

Business Law

3b
INSTRUCTIONS TO CANDIDATES
Read this page before you look at the questions

FBLW
18 November 2002 Day 1 early afternoon

THIS QUESTION PAPER BOOKLET IS ALSO YOUR ANSWER BOOKLET. Sufficient space has been provided for you to write your answers and also for workings where questions require them. For section B questions, you must write your answers in the shaded space provided. Please note that you will NOT receive marks for your notes or workings. You are allowed two hours to answer this question paper. All questions are compulsory. Answer the ONE question in section A (this has 25 sub-questions and is on pages 2-8) Answer the THREE questions in section B (these are on pages 9-14) You are advised to spend 10 minutes reading through the paper before starting to answer the questions. You should spend no more than 55 minutes on answering the ONE question in section A, which has 25 sub-questions. You should spend no more than 55 minutes on answering the THREE questions in section B. You are advised that, in section B questions, you will receive no marks if you exceed the word limits indicated. Hand this entire booklet to the invigilators at the end of the examination. You are NOT permitted to leave the examination hall with this booklet. Do NOT write your name or your student registration number anywhere on this booklet.

The Chartered Institute of Management Accountants 2002

FOR FREE CIMA, ACCA & CAT RESOURCES VISIT: http://kaka-pakistani.blogspot.com

SECTION A 50 MARKS ANSWER ALL TWENTY-FIVE SUB-QUESTIONS 2 MARKS EACH


Each of the sub-questions numbered from 1.1 to 1.25 inclusive, given below, has only ONE correct answer. REQUIRED: Place a circle O around the letter A, B, C or D that gives the correct answer to each sub-question. If you wish to change your mind about an answer, block out your first answer completely and then circle another letter. You will NOT receive marks if more than one letter is circled. Please note that you will NOT receive marks for any workings to these sub-questions. Question One 1.1 Which of the following is correct? (i) (ii) (iii) Auditors who provide negligent advice may be held liable for breach of contract by the company which appointed them. Auditors who provide negligent advice to the company which appointed them may be held liable for breach of contract by the company and its shareholders. Auditors who provide advice to a particular person and who know what the advice will be used for may be held liable to that person in the tort of negligence if the advice proves to be incorrect and was carelessly prepared.

A B C D

(i) only. (i) and (ii) only. (i) and (iii) only. (ii) and (iii) only.

1.2 A B C D

Which ONE of the following is correct? All English courts must apply European Law even if it contradicts English Law. If European and English Law conflict, English courts cannot apply either law. An English court must apply English Law unless it obtains the government's permission to apply European Law. All English courts are obliged to apply English Law even if it contradicts European Law.

FBLW

November 2002

FOR FREE CIMA, ACCA & CAT RESOURCES VISIT: http://kaka-pakistani.blogspot.com

1.3

Which of the following is correct? (i) (ii) (iii) A contract is entered into voluntarily whereas criminal offences are imposed by the state. A tort is entered into voluntarily whereas criminal offences are imposed by the state. A contract is entered into voluntarily whereas a tort is imposed by the state. B (ii) only. C (i) and (iii) only. D (ii) and (iii) only.

(i) only.

1.4

Which of the following is correct? (i) (ii) (iii) A wrongful dismissal cannot also be an unfair dismissal. An unfair dismissal can also be a wrongful dismissal. An unfair dismissal must also be a wrongful dismissal. B (ii) only. C (ii) and (iii) only. D (iii) only

(i) only.

1.5

Brian has been employed by Wye Ltd for 10 years. His contract of employment states that if either Wye Ltd or Brian wishes to terminate the contract, each party must give the statutory minimum period of notice.

Which ONE of the following is correct? A B C D Both Brian and Wye Ltd are entitled to 10 weeks notice. Brian is entitled to 10 weeks notice but Wye Ltd is entitled to only 1 weeks notice. Brian is entitled to 1 months notice and Wye Ltd is entitled to 10 weeks notice. Both Brian and Wye Ltd are entitled to 1 weeks notice.

1.6

Charles recently purchased some goods at an auction sale.

Which of the following is correct? (i) (ii) (iii) A The contract was concluded by the fall of the Auctioneers hammer. The Auctioneers call for bids was an invitation to treat. Once the bidding had started, the Auctioneer was unable to withdraw the goods from the sale. B (i) and (ii) only. C (ii) and (iii) only. D (i), (ii) and (iii).

(i) only.

November 2002

FBLW

FOR FREE CIMA, ACCA & CAT RESOURCES VISIT: http://kaka-pakistani.blogspot.com

1.7

Tee Ltd placed some computers in its shop window with a notice which read:

Special offer. Internet-ready computers for sale at 400.


Which of the following is correct? (i) (ii) (iii) A The notice amounts to an invitation to treat. When Anne called in to the shop and offered 350 for one of the computers, she had made a counter offer. Tee Ltd is obliged to sell a computer to anyone who can pay the price. B (i) only. C (i), (ii) and (iii). D (ii) only.

(i) and (ii) only.

1.8

Which of the following is correct? (i) (ii) (iii) A contract of guarantee is unenforceable unless it is evidenced in writing. A contract to sell land must be in writing. A contract of employment must be in writing. B (i) and (iii) only. C (ii) and (iii) only. D (i), (ii) and (iii).

(i) and (ii) only.

1.9

Which of the following is INCORRECT? (i) (ii) (iii) Legal title cannot pass on a contract induced by misrepresentation. Misrepresentation renders a contract voidable. A victim of negligent misrepresentation may seek rescission and damages. B (ii) only. C (i) and (iii) only. D (ii) and (iii) only.

(i) only.

1.10 In which of the following is there a presumption that legal relations are intended? A B C D A promise by a father to make a gift to his son. A commercial transaction. A domestic arrangement. A social arrangement.

1.11 Which ONE of the following is correct? A B C D A contract is frustrated when something happens after it has been entered into which renders the contract more difficult to perform. A contract is frustrated when a party expressly agrees to manufacture and supply goods and then discovers that they will be far more expensive to produce than he thought at the time of the contract. A contract is frustrated when something happens after it has been entered into which renders the contract impossible to perform. A contract is frustrated if it is impossible to perform at the time that it is made.

FBLW

November 2002

FOR FREE CIMA, ACCA & CAT RESOURCES VISIT: http://kaka-pakistani.blogspot.com

1.12 Exe Ltd was under contract to deliver goods by road to London for Wye Ltd for 2,000. After part of the journey was completed, the delivery vehicle broke down and Wye Ltd was forced to arrange for Zed Ltd to complete the delivery. If there are no provisions in the contract to deal with this situation, which of the following is correct? A B C D Exe Ltd is entitled to part of the delivery fee. Exe Ltd is entitled to nothing. Exe Ltd is entitled to a reasonable sum for the work done. Exe Ltd is entitled to the full 2,000.

1.13 A Ltd contracted to deliver a quantity of goods to B Ltd for 5,000. The goods were delivered and A Ltd submitted an invoice to B Ltd for the amount due which contained a number of new terms. Which of the following is INCORRECT? A B C D The invoice is a contractual document and B Ltd is bound by the terms on the invoice. B Ltd is only bound by the terms if it was given notice of them at or before the time of the contract. B Ltd is bound by the terms if there is a sufficient course of dealings between A Ltd and B Ltd so that B Ltd is assumed to know of the terms. If B Ltd is unaware of the terms, it can only be bound by them if it agrees to be so.

1.14 ABC Ltd has contracted with DEF Ltd. If ABC Ltd acts in breach of a warranty, which of the following is correct? (i) (ii) (iii) A DEF Ltd may terminate the contract and sue for damages. DEF Ltd may sue for damages but may not terminate the contract. DEF Ltd may ignore the breach and continue with the contract. B (i) and (iii) only. C (ii) and (iii) only. D (i), (ii) and (iii).

(i) only.

1.15 Which ONE of the following remedies is NOT available for a breach of a contract to provide personal services? A B C D Damages. A decree of specific performance. An injunction. An action for the price.

November 2002

FBLW

FOR FREE CIMA, ACCA & CAT RESOURCES VISIT: http://kaka-pakistani.blogspot.com

1.16 Which of the following is INCORRECT? (i) (ii) (iii) A A liquidated damages clause will be void if it amounts to a penalty clause. A liquidated damages clause will apply where it is a genuine attempt to pre-estimate the loss caused by a breach of contract. A liquidated damages clause cannot be valid if it is for an amount in excess of the actual loss caused by the breach of contract. B (ii) only. C (i), (ii) and (iii). D (iii) only.

(i) and (ii) only.

1.17 Which of the following is INCORRECT in relation to the remedy of rescission? (i) (ii) (iii) A Rescission is available automatically in the event of a breach of contract. The victim of a misrepresentation may automatically claim rescission. Rescission is designed to return the parties to their pre-contract position. B (ii) only. C (i) and (ii) only. D (ii) and (iii) only.

(i) only.

1.18 Quentin was employed by Bee Ltd as its senior design consultant. Quentin contracted with Bee Ltd that when his employment with the company ceased, he would not act in competition with it or solicit its customers. After Quentin left Bee Ltd, he registered a company called Cee Ltd, which immediately began working in competition with Bee Ltd and soliciting its customers. Bee Ltd complained to Quentin about this conduct, but Quentin stated that as the work was being undertaken by Cee Ltd, a separate legal entity, he had not acted in breach of the agreement with Bee Ltd. Which ONE of the following is correct? A B C D Quentin is correct and cannot be said to be in breach of the agreement with Bee Ltd. If the court finds that Cee Ltd was set up by Quentin to avoid the agreement with Bee Ltd, it will lift the corporate veil and enforce Quentins contract with Bee Ltd against Quentin and Cee Ltd. Cee Ltd will be bound by the agreement because a company is always liable for the actions of its shareholders. The agreement between Bee Ltd and Quentin is of no legal effect as it attempts to regulate Quentins activities after he has left Bee Ltds employment.

1.19 The Memorandum of Dee Ltd, a property development company, states that the company has power to borrow in furtherance of its objects and that the directors have authority to borrow up to 200,000. The board has resolved to purchase a piece of land for 300,000. The Midwest Bank plc has agreed to make a loan of 250,000 to Dee Ltd to acquire the land. Which ONE of the following is correct? A B C D The loan is void as Dee Ltd has acted ultra vires. As the directors have exceeded their authority, the bank cannot enforce the loan against Dee Ltd. As the directors have resolved to obtain the loan, the transaction is lawful. The loan is ultra vires the directors who will be personally liable for any loss caused to the company unless their actions are ratified by the shareholders.
6 November 2002

FBLW

FOR FREE CIMA, ACCA & CAT RESOURCES VISIT: http://kaka-pakistani.blogspot.com

1.20 Which of the following is correct? (i) A public company cannot commence trading until it has received a certificate from the Registrar of Companies confirming that it has satisfied the minimum requirements as to authorised and issued share capital. If a public company commences trading without a trading certificate, and fails to meet its obligations, the directors may be held jointly and severally liable for those obligations. A private company may commence trading upon receipt of its certificate of incorporation. B (ii) and (iii) only. C (i) and (iii) only. D (i), (ii) and (iii).

(ii) (iii) A

(i) and (ii) only.

1.21 Which ONE of the following is correct? A B C D The shares of all public limited companies are quoted on the Stock Exchange. The company secretary of a public limited company must be qualified. A private limited company must have at least two shareholders. A public limited company cannot trade until it has paid-up share capital of at least 50,000.

1.22 Which ONE of the following may requisition an extraordinary general meeting? A B C D Members holding not less than one-twentieth of the companys issued share capital which carries voting rights. Members holding not less than one-tenth of the companys issued share capital which carries voting rights. Members holding not less than one-quarter of the companys issued share capital which carries voting rights. Members holding not less than one-half of the companys issued share capital which carries voting rights.

1.23 What type of resolution is needed if a private company wishes to reduce, to 90%, the percentage needed to authorise the provision of short notice? A Elective. B Extraordinary. C Ordinary. D Special.

November 2002

FBLW

FOR FREE CIMA, ACCA & CAT RESOURCES VISIT: http://kaka-pakistani.blogspot.com

1.24 PRS Ltd wishes to use its assets as security for a loan to be made by B Bank plc to Peter, a shareholder in PRS Ltd, so that he may purchase more of the companys share capital. Which ONE of the following is correct? A B C D The proposal is lawful, if approved by a special or written resolution and the correct procedure is followed. The proposal is lawful, if the company passes an ordinary resolution and the correct procedure is followed. The proposal is lawful, only if it is approved by the court and the correct procedure is followed. The proposal is unlawful, as a company cannot give financial assistance for the purchase of its own shares.

1.25 Which ONE of the following correctly describes the maintenance of capital principle? A B C D A company cannot use its share capital which must be set aside as a fund for creditors. In general, a company can use its share capital for any reason whatsoever, as long as it ensures that there is sufficient remaining to meet the companys debts. In general, a company must use its capital for the purposes set out in its objects clause and cannot return it to its members. A company cannot use its share capital and may only return it to its members with the permission of the court.

Total = 50 Marks

FBLW

November 2002

FOR FREE CIMA, ACCA & CAT RESOURCES VISIT: http://kaka-pakistani.blogspot.com

SECTION B 50 MARKS ANSWER ALL THE QUESTIONS


Question Two A Ltd contracted to install computerised security systems at each of B plc's 100 shops by 30 June 2002 for a total price of 200,000. The contract between A Ltd and B plc contained the following clause: "A Ltd hereby agrees to pay B plc the sum of 1,000 per day by way of liquidated damages, in respect of each unfinished shop for each day after 30 June 2002." A Ltd sub-contracted with C Ltd which agreed to install the systems at 50 of B plc's shops by 30 June 2002. On 1 February 2002, however, C Ltd wrote to A Ltd to explain that because of financial problems, the company would be unable to complete the contract by 30 June 2002. On 4 February 2002, A Ltd agreed to lend C Ltd 15,000 secured by a floating charge over C Ltd's stock. In addition, A Ltd promised to pay C Ltd a bonus of 5,000 if the company completed the work on time. Following this sequence of events, the following facts have emerged: (i) C Ltd completed the work on time, but was placed in insolvent liquidation on 1 July 2002. A Ltd has refused to pay the bonus of 5,000. A Ltd completed its contract with B plc, but was four days late in finishing three of the shops. As a result, B plc has advised A Ltd that the price of 200,000 will be reduced by 12,000 in accordance with the above liquidated damages provision.

(ii)

Do not write in these columns below


For use by the first marker For use by the second marker

Required (a) Delete as appropriate * and complete this sentence:


In this case, A Ltd is* / is not* liable to pay C Ltd the bonus of 5,000 because

Marks available 1

. (Maximum of 30 words) (b) State the three characteristics of a floating charge (i) (ii) (iii)
(Maximum of 10 words for each of the three sentences) 3 Sub-total: 7

Requirements (c) (e) of question two are on page 9

November 2002

FBLW

FOR FREE CIMA, ACCA & CAT RESOURCES VISIT: http://kaka-pakistani.blogspot.com

Question Two continued Do not write in these columns below For use For use by the by the Marks first second available marker marker

Required (c) Complete this sentence:

A Ltd will be entitled to recover its loan from the proceeds of the sale of C Ltd's stock, as long as there is sufficient remaining after payment of the .
(Maximum of 2 words)

(d) Delete as appropriate * and complete this sentence:


B plc can* / cannot* rely on the above liquidated damages clause to reduce A Ltd's fee by 12,000 because
1

. (Maximum of 30 words)

(e)

Complete this sentence: If it could be shown that C Ltd had lied about its financial difficulties, in order to pressurise A Ltd into promising to pay a bonus, then A Ltd would not be liable to pay the bonus of 5,000 because

. (Maximum of 30 words)

4 Sub-total: 10

Total Marks for Question Two =

17

Total for Question Two = 17 Marks


Space for notes for Question Two

FBLW

10

November 2002

FOR FREE CIMA, ACCA & CAT RESOURCES VISIT: http://kaka-pakistani.blogspot.com

Question Three Zed plc has decided to carry out the following: (i) (ii) To appoint Michelle as the companys managing director. To borrow 5 million from Exe Bank plc secured by a fixed charge over the companys land and a floating charge over the companys stock. To acquire Franks business by issuing to him 200,000 ordinary 1 shares in Zed plc. To purchase a number of its own shares on the Stock Exchange. Do not write in these columns below
For use by the first marker For use by the second marker

(iii) (iv)

Required (a) Complete these sentences:


Michelle may be appointed managing director by .
(Maximum of 3 words)

Marks available

The appointment of all the other directors must be approved by the .


(Maximum of 3 words)

(b) Complete these sentences: The fixed and floating charges must be registered with the Registrar of Companies within days. (State the number of days)
If they are not registered, they will be against the liquidator, administrator or any creditor of the company.
(Maximum of 1 word)

(c) Delete as appropriate * and complete this sentence:


The 200,000 ordinary 1 shares to be issued by Zed plc must* / need not* be offered to Zed plcs existing shareholders first because
1

. (Maximum of 30 words)

3 Sub-total: 9

Space for notes for Question Three

Requirements (d) and (e) of question three are on page 12


November 2002 11 FBLW

FOR FREE CIMA, ACCA & CAT RESOURCES VISIT: http://kaka-pakistani.blogspot.com

Question Three continued Do not write in these columns below For use For use by the by the Marks first second available marker marker

Required (d) Delete as appropriate * and complete this sentence:


Before Franks business can be acquired by Zed plc, it must* / need not* be valued by the auditor because Zed plc is a .
(Maximum of 3 words)

1 1

(e) Delete as appropriate * and complete the following sentences:


The types of fund which must be used by Zed plc to purchase its own shares are
(Maximum of 3 words) 1 1 1 1 Sub-total: 6 Total Marks for Question Three = 15

or

.
(Maximum of 3 words)

As this is an/a off market* / market* purchase, it must be authorised by resolution. (Maximum of 2 words)

Total for Question Three = 15 Marks

Space for notes for Question Three

FBLW

12

November 2002

FOR FREE CIMA, ACCA & CAT RESOURCES VISIT: http://kaka-pakistani.blogspot.com

Question Four Michelle, Nicola, Owen and Paul have decided to go into business together selling mobile telephones through a company limited by shares. A company is to be incorporated called MNOP Ltd and each person is to subscribe for 25,000 ordinary 1 shares at a cost of 50,000. Michelle, Nicola, Owen and Paul have agreed the following: (i) (ii) Each is to be appointed as an executive director of MNOP Ltd on a fixed term contract for six years. The Memorandum of Association of the company is to be worded to allow the company to diversify into any trade or business which the members may wish to carry on in the future. The Articles of Association of MNOP Ltd will include the following provision: "Any member who wishes to sell his or her shares must offer them to the other members who shall purchase them at a fair price as determined by the companys auditor." Do not write in these columns below
For use by the first marker For use by the second marker

(iii)

Required (a) Complete this sentence:


As Michelle, Nicola, Owen and Paul are to have service contracts for a period exceeding years (State the number of years) on terms that they cannot be dismissed by reasonable notice, the contracts will need to be approved by the shareholders passing
(Maximum of 2 words) or

Marks available

1 1

resolution.
(Maximum of 2 words)

(b) Complete this sentence:


To enable the company to diversify into any trade or business, the clause (Maximum of 1 word) should include a statement that
1

.
(Maximum of 20 words)

2 Sub-total: 6

Requirements (c) (e) of question four are on page 14

November 2002

13

FBLW

FOR FREE CIMA, ACCA & CAT RESOURCES VISIT: http://kaka-pakistani.blogspot.com

Question Four continued Do not write in these columns below

Required (c) Delete as appropriate * and complete this sentence:


The shareholders of MNOP Ltd will * / will not * be bound by the clause in the companys Articles of Association concerning the sale of shares because

Marks available

For use by the first marker

For use by the second marker

. (Maximum of 30 words)

(d) Delete as appropriate * and complete these sentences:


As the 100,000 ordinary 1 shares have been issued for a total price of 200,000, a total of (State the amount) must be transferred to a account. (Maximum of 2
words) The amount held in that account may* / may not* be used to 1 1 1

pay a dividend.

(e) Delete as appropriate * and complete the following sentences:


Michelle, Nicola, Owen and Paul have each agreed to work under a contract (Maximum of 2 words) which means that they are employees* / independent contractors* of the company. It follows that if any one of the directors is dismissed before the expiry of 12 months he or she will* / will not* be entitled to sue the company for dismissal.
(Maximum of 2 words) Sub-total: 12 Total Marks for Question Four = 18 1 2 1 1

Total for Question Four = 18 Marks

End of Question Paper

FBLW

14

November 2002

FOR FREE CIMA, ACCA & CAT RESOURCES VISIT: http://kaka-pakistani.blogspot.com

Examination Question and Answer Book


Write here your full examination number Centre Code: Hall Code: Desk Number:

Foundation Level

Business Law

3b
INSTRUCTIONS TO CANDIDATES
Read this page before you look at the questions

FBLW
19 May 2003 Monday early afternoon

THIS QUESTION PAPER BOOKLET IS ALSO YOUR ANSWER BOOKLET. Sufficient space has been provided for you to write your answers and also for workings where questions require them. For section B questions, you must write your answers in the shaded space provided. Do not exceed the stated number of words. Please note that you will NOT receive marks for your notes or workings. Do NOT remove any sheets from this booklet: cross through neatly any work that is not to be marked. Avoid the use of correction fluid. You are allowed two hours to answer this question paper. All questions are compulsory. Answer the ONE question in section A (this has 25 sub-questions and is on pages 2-10) Answer the THREE questions in section B (these are on pages 12-17) You are advised to spend 10 minutes reading through the paper before starting to answer the questions. You should spend no more than 55 minutes on answering the ONE question in section A, which has 25 sub-questions. You should spend no more than 55 minutes on answering the THREE questions in section B. You are advised that, in section B questions, you will receive no marks if you exceed the word limits indicated. Hand this entire booklet to the invigilators at the end of the examination. You are NOT permitted to leave the examination hall with this booklet. Do NOT write your name or your student registration number anywhere on this booklet.

The Chartered Institute of Management Accountants 2003

FOR FREE CIMA, ACCA & CAT RESOURCES VISIT: http://kaka-pakistani.blogspot.com

SECTION A 50 MARKS ANSWER ALL TWENTY-FIVE SUB-QUESTIONS 2 MARKS EACH


Each of the sub-questions numbered from 1.1 to 1.25 inclusive, given below, has only ONE correct answer. REQUIRED: Place a circle O around the letter A, B, C or D that gives the correct answer to each sub-question. If you wish to change your mind about an answer, block out your first answer completely and then circle another letter. You will NOT receive marks if more than one letter is circled. Please note that you will NOT receive marks for any workings to these sub-questions. Question One 1.1 A B C D Which ONE of the following is the superior source of English law? A precedent established by the House of Lords. A provision in an Act of Parliament. A government pronouncement. A Statutory Instrument.

1.2 A B C D

Which ONE of the following most accurately describes the ratio decidendi? It is a statement of law by a court which is binding on all higher courts which are called upon to decide a similar case. It is a statement of law by a court which is not binding on lower courts which are called upon to decide a similar case. It is a statement of law by a court which is binding on all lower courts which are called upon to decide a similar case. It is a statement of law by a court which is binding on all future courts which are called upon to decide a similar case.

FBLW

May 2003

FOR FREE CIMA, ACCA & CAT RESOURCES VISIT: http://kaka-pakistani.blogspot.com

1.3

Which of the following statements are CORRECT? (i) (ii) (iii) To succeed in the tort of negligence a claimant must prove that the defendant owed him/her a duty of care. To succeed in the tort of negligence the claimant must prove that the defendant has broken a duty to take care. To succeed in the tort of negligence a claimant must prove he/she suffered loss or harm as a result of the defendants breach of the duty to take care.

A B C D

(i) and (ii) only. (ii) and (iii) only. (i) and (iii) only. (i), (ii) and (iii).

1.4 A B C D

Which ONE of the following cannot be carried out by a simple contract? A contract for the sale of a motor car. A contract of employment. A contract for the sale of land. A contract for the provision of services.

1.5

Jack was induced to enter into a contract with Karl by Karls negligent misrepresentation.

Which of the following are CORRECT? (i) (ii) (iii) A B C D (i) only. (i) and (ii) only. (ii) and (iii) only. (i), (ii) and (iii). Jack has a remedy against Karl in the tort of negligence. Jack has a remedy against Karl under the Misrepresentation Act 1967. Jack has a remedy against Karl in the tort of deceit.

May 2003

FBLW

FOR FREE CIMA, ACCA & CAT RESOURCES VISIT: http://kaka-pakistani.blogspot.com

1.6

In relation to the Unfair Contract Terms Act 1977, which of the following are CORRECT? (i) (ii) (iii) A clause which attempts to exclude liability for death or personal injury caused by negligence is only valid if it is reasonable. A clause which attempts to exclude liability for damage to property is void unless it is reasonable. The Unfair Contract Terms Act applies to contractual terms and to occupiers liability.

A B C D

(i) only. (ii) only. (i) and (iii) only. (ii) and (iii) only

1.7 A B C D

In relation to contractual performance, which ONE of the following is INCORRECT? A person who has substantially performed the contract is entitled to the contract price less so much as is necessary to repair the defects. If one party is prevented from performing the contract by the other party, he or she may claim compensation on a quantum meruit basis. A person who has carried out part of the contract is always entitled to a part of the contract price. A person who has carried out all of his contractual obligations is entitled to all of the contract price.

1.8

Tee Ltd has contracted to use Vee Ltds Grand Hotel for a business conference. Which of the following would be regarded as a valid reason for Vee Ltd for the unavailability of the hotel on the agreed date under the law of frustration? (i) (ii) (iii) The hotel was closed due to flood damage. The hotel was double booked. The hotel manager had arranged to have the hotel redecorated. The decorators had failed to complete the work by the agreed date.

A B C D

(i) only. (iii) only. (ii) and (iii) only. (i), (ii) and (iii).

FBLW

May 2003

FOR FREE CIMA, ACCA & CAT RESOURCES VISIT: http://kaka-pakistani.blogspot.com

1.9

Which of the following entitles the innocent party to cancel the contract? (i) (ii) (iii) A breach of a warranty. A breach of a condition. An express term authorising termination of the contract.

A B C D

(i) only. (i) and (ii) only. (ii) and (iii) only. (i), (ii) and (iii).

1.10 In relation to the remedy of damages, which ONE of the following is INCORRECT? A B C D A victim of a breach of contract may be unable to recover damages if he has failed to mitigate his loss. Mitigation and remoteness of damage are irrelevant where a creditor is suing to recover an agreed debt. A penalty clause in a contract cannot be enforced. Damages cannot be claimed for a breach of a contract to provide personal services.

1.11 In relation to a claim for wrongful dismissal, which of the following statements are INCORRECT? (i) (ii) (iii) (iv) A B C D (i) only. (ii) only. (ii) and (iii) only. (i) and (iv) only. Only employees below the normal retiring age may claim. There is no qualifying period. There is frequently no limit on the amount of compensation that a tribunal can award. Claims must be made within 3 months of the dismissal.

May 2003

FBLW

FOR FREE CIMA, ACCA & CAT RESOURCES VISIT: http://kaka-pakistani.blogspot.com

1.12 Which ONE of the following does not automatically give rise to a claim for unfair dismissal? A B C D Dismissal on grounds of age. Dismissal on grounds of race. Dismissal on grounds of disability. Dismissal on grounds of sex.

1.13 Claire's employment with Wye Ltd began 8 years ago. Her contract of employment states that she may be required to work at any of the company's offices in London, Birmingham or Manchester. Claire has for the entire 8 years worked at the company's London office. The company has now asked Claire to move to its Manchester office. Which ONE of the following statements is CORRECT? A B C D Claire cannot be required to move to the Manchester office. Claire's contract contains a mobility clause which permits her employer to require her to work at any of the company's offices. The fact that Claire has always worked in the London office prohibits her employer from requiring her to move to any of the company's other offices. If Claire is required to move to the Manchester office, she may make a claim for automatic unfair and wrongful dismissal.

1.14 A business operates under the name "High Street Furnishings. Which ONE of the following types of business organisation must this be? A B C D A partnership. A sole tradership. A private limited company. Any type of business organisation as "High Street Furnishings" is a business name.

FBLW

May 2003

FOR FREE CIMA, ACCA & CAT RESOURCES VISIT: http://kaka-pakistani.blogspot.com

1.15 Which of the following are CORRECT? (i) (ii) (iii) A company is owned by its shareholders and managed by its directors. A company is entitled to own property in its own name. If business is carried on through a company limited by shares, the shareholders can never incur personal liability over and above the amount due on their shares even if the veil of incorporation is lifted.

A B C D

(i) only. (i) and (ii) only. (i) and (iii) only. (i), (ii) and (iii).

1.16 Which of the following statements are INCORRECT? (i) (ii) (iii) A B C D (i) only. (ii) only. (i) and (iii) only. (i), (ii) and (iii). A public limited company may commence trading as soon as it receives a certificate of incorporation. A private company may re-register as a public company by passing a special resolution. A public company cannot pass an elective resolution.

1.17 Which of the following is a consequence of corporate personality? (i) (ii) (iii) A B C D (i) only. (i) and (ii) only. (ii) and (iii) only. (i), (ii) and (iii). The company is fully liable for its own debts. A shareholder has limited liability. The general rule is that if a wrong has been done to the company, the company alone is entitled to sue.

May 2003

FBLW

FOR FREE CIMA, ACCA & CAT RESOURCES VISIT: http://kaka-pakistani.blogspot.com

1.18 Which ONE of the following resolutions does not generally have to be filed at Companies House? A B C D Ordinary. Special. Extraordinary. Elective.

1.19 In relation to the Annual General Meeting (AGM), which ONE of the following is INCORRECT? A B C D So long as a company holds its first AGM within 18 months of its incorporation, it need not hold it in the year of its incorporation or in the following year. In general, there must not be a gap of more than 15 months between AGMs. In general, notice of 21 clear days must be given to the members of the intention to hold an AGM. A majority in number of members holding at least 75% of the issued share capital may agree to less than 21 clear days' notice for an AGM.

1.20 In order for a private company to be able to purchase its own shares out of capital, the directors must make a statutory declaration containing a statement that, in their opinion, the company will be able to carry on business as a going concern and will be able to pay its debts as they fall due for a period of A B C D 6 months. 12 months. 18 months. 24 months.

1.21 Which of the following would be regarded as an issue of shares for an improper purpose? (i) (ii) (iii) A B C D (i) only. (i) and (ii) only. (ii) and (iii) only. (iii) only. An issue of shares in return for a non-cash consideration. An issue of shares to enable the directors to maintain control of the board. An issue of shares to prevent a take-over bid.

FBLW

May 2003

FOR FREE CIMA, ACCA & CAT RESOURCES VISIT: http://kaka-pakistani.blogspot.com

1.22 Which of the following are CORRECT? (i) (ii) (iii) A board resolution to borrow money is binding on the company. A decision by the managing director to borrow money is binding on the company. The shareholders may veto any decision to borrow money by passing an ordinary resolution.

A B C D

(i) only. (i) and (ii) only. (ii) and (iii) only. (i), (ii) and (iii).

1.23 Which of the following are INCORRECT? (i) (ii) (iii) The first directors are appointed by a resolution of the members at the companys first Annual General Meeting. A director is a person who occupies the position of director by whatever name he or she is called. A shadow director is a person in accordance with whose directions or instructions the directors of the company are accustomed to act.

A B C D

(i) only. (i) and (ii) only. (ii) and (iii) only. (i), (ii) and (iii).

1.24 Section 459 of the Companies Act 1985 empowers the court to grant a remedy in the event of a companys affairs being conducted in a manner unfairly prejudicial to the interests of its members. Who may petition for an order under this Section? (i) (ii) (iii) A B C D (i) only. (i) and (ii) only. (ii) and (iii) only. (iii) only. Any member. The holders of at least 10% of the companys issued share capital. The directors.

May 2003

FBLW

FOR FREE CIMA, ACCA & CAT RESOURCES VISIT: http://kaka-pakistani.blogspot.com

1.25 The directors of a company limited by shares owe a duty to have regard to the interests of the companys employees in general, as well as the interests of its members. Who may enforce this duty? A B C D The company. The employees. The shareholders. The directors.

Total = 50 Marks

End of Section A

FBLW

10

May 2003

FOR FREE CIMA, ACCA & CAT RESOURCES VISIT: http://kaka-pakistani.blogspot.com

SECTION B 50 MARKS ANSWER ALL THREE QUESTIONS


Question Two
Edward, who is a self-employed builder, has experienced the following difficulties. (i) He contracted to build a conservatory for Fiona at a total cost of 15,000. On completion of the job, however, Fiona refused to pay the amount due, despite the fact that she had no complaints as to the quality of the workmanship. Instead, Fiona sent a letter to Edward stating that she had decided that the price was too high and that she was enclosing a cheque for 10,000 in full and final settlement of the account. At first, Edward agreed to accept the amount and cashed the cheque. Later, however, he changed his mind and decided to commence legal action against Fiona to recover the balance. He contracted to carry out some building work for George for 8,000. After approximately half the work had been completed, Tom, who had worked for Edward for only 8 months, negligently caused some damage to Georges property. As a result, George refused to allow Edward to complete the work despite the fact that Tom was instantly dismissed. In March 2003, he contracted to build an extension for Henry commencing in December 2003. In April 2003, Henry advised Edward that he had sold the house and no longer required the extension to be built. Do not write in these columns below
For use Marks by the first available marker For use by the second marker

(ii)

(iii)

Required: (a) Delete as appropriate * and complete this sentence:


At common law, Edward can* /cannot* sue Fiona to recover the balance due of 5,000 because

. (Maximum of 30 words) (b) Delete as appropriate * and complete this sentence: Edward will* /will not* be liable for the damage caused by Tom to Georges property because

. (Maximum of 30 words) Sub-total:

3 8

May 2003

11

FBLW

FOR FREE CIMA, ACCA & CAT RESOURCES VISIT: http://kaka-pakistani.blogspot.com

Question Two continued


Do not write in these columns below
For use Marks by the first available marker For use by the second marker

Required: (c) Delete as appropriate and complete this sentence:


Edward will* /will not* be entitled to claim compensation from George in respect of the work carried out for him because

3 . (Maximum of 30 words)

(d) Complete this sentence:


Tom will have a claim against Edward for (Maximum 1 word) dismissal, but not for (Maximum 1 word) dismissal because 1 1

2 . (Maximum of 20 words)

(e) Delete as appropriate and complete this sentence:


By cancelling the contract before it was due to be carried out, Henry has acted in
(Maximum 1 word) breach of contract.

1 1

Edward will* /will not* have to wait until after December 2003 before he can sue for breach of contract.

Total for Question Two = 18 Marks

FBLW

12

May 2003

FOR FREE CIMA, ACCA & CAT RESOURCES VISIT: http://kaka-pakistani.blogspot.com

Question Three
Adam, Ben and Chris are the only directors and shareholders in ABC Ltd, each holding 100,000 ordinary 1 shares. The company is a retailer of computers and has traded very successfully. However, a large supermarket chain began to sell computers at prices well below those charged by ABC Ltd and, as a result, sales fell and indebtedness increased. An emergency board meeting was called on 1 April 2002 to discuss ABC Ltds problems. The meeting was attended by the companys auditor who advised that insolvency appeared inevitable. The board unanimously agreed to the following: (i) (ii) The company would continue trading for a further twelve months in the hope that matters would improve. Adam (who had already provided the company with an unsecured loan of 10,000) would lend a further 20,000, subject to the company providing him with security in the form of a floating charge in respect of "all money" owed to him by the company. Ben would provide the company with an unsecured loan of 15,000 which the company would repay in full on 1 April 2003.

(iii)

Unfortunately, matters did not improve and the company continued to suffer financial difficulties. The company repaid Bens loan of 15,000 on 10 January 2003 and the company was placed in insolvent liquidation on 15 March 2003. At the date of liquidation, Adam was, in fact, owed 35,000 by the company, as he had made loans of 25,000 following the issue of the floating charge. Do not write in these columns below

Required:
Marks available

For use by the first marker

For use by the second marker

(a) Complete this sentence in the shaded area below:


The directors may be caught by the wrongful trading provisions of the Insolvency Act 1986 because

(Maximum of 30 words)

(b) Complete this sentence:


As a result of the wrongful trading provisions, the directors may be
(Maximum 2 words) for some

of the companys debts, even though they have been trading on the companys behalf.

(c) Complete this sentence:


Because ABC Ltd has issued a floating charge to a connected person and gone into insolvent liquidation within the relevant time limit of months, Adam may rely on the charge for . Sub-total: , 1 and 2 2 11 but must stand as an unsecured creditor for

May 2003

13

FBLW

FOR FREE CIMA, ACCA & CAT RESOURCES VISIT: http://kaka-pakistani.blogspot.com

Required:
Marks available

For use by the first marker

For use by the second marker

(d) Complete these sentences:


The repayment of Bens loan is likely to be challenged by the
(Maximum one word) as a preference. Because

1 2

Ben is a

, (Maximum 2 words)

it will be presumed that the company was influenced by a desire to prefer him.

Total for Question Three = 14 Marks

FBLW

14

May 2003

FOR FREE CIMA, ACCA & CAT RESOURCES VISIT: http://kaka-pakistani.blogspot.com

Question Four
Lucy, Mike, Neil and Owen are the only directors and shareholders in LMNO Ltd, each holding 25% of the companys issued share capital. The company carries on business as a wholesaler of books. (i) The board has unanimously decided that a number of amendments need to be made to the constitution and administration of the company. The Articles of Association are to be amended and the company is to dispense with the need to hold an AGM. The contract of employment of Pat, the Senior Sales Manager of LMNO Ltd, included a provision that if she left the company she would not compete with it for a period of 12 months from the date of leaving within the normal area of the companys operations. Pat left, set up a company called P (Books) Ltd, and, on behalf of the company, sent advertising materials to all LMNO Ltd customers.

(ii)

LMNO Ltd owns 15% of the shares in PQR Ltd, which owned a book retailing business. The board of LMNO Ltd wrote to the auditors of PQR Ltd explaining, in confidence, that LMNO Ltd was considering making an offer for all the issued share capital of PQR Ltd and would like confirmation, therefore, that the most recent audit report provided an accurate picture of PQR Ltds finances. The auditor replied and provided the necessary confirmation. As a result, LMNO Ltd acquired the entire share capital of PQR Ltd. The board of LMNO Ltd now believes that the audit report was carelessly Do not write in these prepared and this resulted in PQR Ltds assets being overvalued and columns below LMNO Ltd paying far too much for the companys shares.
For use Marks by the first available marker For use by the second marker

Required: (a) Complete this sentence:


In order to alter the Articles of Association, the shareholders need to pass a (Maximum 1 word) or resolution. (Maximum 1 word). The resolution must be filed at Companies House within days of the date the resolution was passed.

1 1 1

(b) Delete as appropriate * and complete these sentences:


To remove the need to hold an AGM,
(Maximum 2 words) resolution must be passed. This type of resolution must be agreed to by % of the shareholders and the

1 1 1

auditor must* /need not* be notified.

(c) Delete as appropriate * and complete these sentences:


By competing with LMNO Ltd, Pat has acted
(Maximum 4 words) and the

1 1 and 1 1 1

company will* /will not* be entitled to obtain


(Maximum 2 words) to prevent further competition.

LMNO Ltd is* / is not* entitled to take action against Pat* / Pat and P (Books) Ltd* because

. (Maximum of 30 words) sub-total:

3 14

May 2003

15

FBLW

FOR FREE CIMA, ACCA & CAT RESOURCES VISIT: http://kaka-pakistani.blogspot.com

Required: (d) Delete as appropriate * and complete this sentence:


LMNO Ltd will* /will not* be able to sue the auditor of PQR Ltd for compensation because

For use Marks by the first available marker

For use by the second marker

. (Maximum of 30 words)

Total for Question Four = 18 Marks

End of Question Paper

FBLW

16

May 2003

FOR FREE CIMA, ACCA & CAT RESOURCES VISIT: http://kaka-pakistani.blogspot.com

Examination Question and Answer Book


Write your full examination number, your contact ID and your name on a double-sided card, which must be attached to this booklet here.

Foundation Level

Business Law

3b
INSTRUCTIONS TO CANDIDATES
Read this page before you look at the questions

FBLW
17 November 2003 Monday early afternoon

THIS QUESTION PAPER BOOKLET IS ALSO YOUR ANSWER BOOKLET. Sufficient space has been provided for you to write your answers and also for notes where questions require them. For section B questions, you must write your answers in the shaded space provided. Please note that you will NOT receive marks for your notes. Do NOT remove any sheets from this booklet: cross through neatly any work that is not to be marked. Avoid the use of correction fluid. You are allowed two hours to answer this question paper. All questions are compulsory. Answer the ONE question in section A (this has 25 sub-questions) Answer the THREE questions in section B You are advised to spend 10 minutes reading through the paper before starting to answer the questions. You should spend no more than 55 minutes on answering the ONE question in section A, which has 25 sub-questions. You should spend no more than 55 minutes on answering the THREE questions in section B. You are advised that, in section B questions, you will receive no marks if you exceed the word limits indicated. Hand this entire booklet to the invigilators at the end of the examination. You are NOT permitted to leave the examination hall with this booklet. Do NOT write your name or your contact ID anywhere on this booklet.

The Chartered Institute of Management Accountants 2003

FOR FREE CIMA, ACCA & CAT RESOURCES VISIT: http://kaka-pakistani.blogspot.com

SECTION A 50 MARKS ANSWER ALL TWENTY-FIVE SUB-QUESTIONS 2 MARKS EACH


Each of the sub-questions numbered from 1.1 to 1.25 inclusive, given below, has only ONE correct answer. REQUIRED: Place a circle O around the letter A, B, C or D that gives the correct answer to each sub-question. If you wish to change your mind about an answer, block out your first answer completely and then circle another letter. You will NOT receive marks if more than one letter is circled.

Question One
1.1 Which of the following statements is INCORRECT? (i) (ii) (iii) A B C D (i) only. (ii) only. (i) and (iii) only. (ii) and (iii) only. Common law and equitable principles can be applied by all the courts. A wrong committed by one person cannot result in both civil and criminal liability. To be found guilty, the prosecution must prove beyond reasonable doubt that the accused committed the offence.

1.2

Which of the following statements is CORRECT? (i) (ii) (iii) A contract is entered into voluntarily, whereas criminal offences are imposed by the state. A contract is entered into voluntarily, whereas a tort is imposed by the state. A tort is entered into voluntarily, whereas criminal offences are imposed by the state.

A B C D

(i) only. (i) and (ii) only. (i) and (iii) only. (ii) and (iii) only.

FBLW

November 2003

FOR FREE CIMA, ACCA & CAT RESOURCES VISIT: http://kaka-pakistani.blogspot.com

1.3

Which ONE of the following statements is CORRECT?

In the tort of negligence, a defendant has broken his/her duty of care if he/she A B C D has failed to take every step to avoid harming the claimant. has failed to take all necessary care. has failed to take all reasonable care. has failed to act to the best of his/her abilities.

1.4

Anne offered to sell her computer to Ben for 600. Which of the following would terminate the offer? (i) (ii) (iii) Before Ben had replied, Anne wrote to Ben withdrawing her offer. Annes letter was not received by Ben. Ben offered Anne 500 for her computer. Anne told Carl that she had changed her mind about selling the computer to Ben. Carl told Ben before he had accepted Annes offer.

A B C D

(i) only. (i) and (iii) only. (ii) and (iii) only. (i), (ii) and (iii).

1.5

Following Carols retirement as a senior employee, Tee plc offered her an "ex gratia payment" of 50,000 "in recognition of Carols excellent service to the company". Carol accepted. (i) (ii) Although there is agreement between Tee plc and Carol, Carol has no contractual right to the payment, as her consideration is past. Although there is agreement between Tee plc and Carol, Carol has no contractual right to the payment because by using the expression "ex gratia", the company is stating that it has no intention to create legal relations. As there is agreement between Tee plc and Carol, the payment is enforceable as any agreement between an employer and an employee is presumed to have legal intent.

Which of the following statements is INCORRECT?

(iii) A B C D (i) only. (ii) only. (iii) only.

(ii) and (iii) only.

November 2003

FBLW

FOR FREE CIMA, ACCA & CAT RESOURCES VISIT: http://kaka-pakistani.blogspot.com

1.6 A B C D

Which ONE of the following statements is INCORRECT? An unreasonable delay by the victim of a misrepresentation in claiming rescission will cause the remedy to be lost. If the victim of a misrepresentation affirms the contract, the remedy of rescission is lost. A victim of a misrepresentation is entitled to rescission even if damages would compensate the victim for his loss. If the victim of a misrepresentation has acted inequitably, the remedy of rescission is lost.

1.7

A Ltd is under contract to manufacture machinery for B Ltd for 200,000.

Which ONE of the following is a valid legal reason for A Ltd failing to complete the contract on time? A B C D A Ltds factory was flooded. A Ltd had taken on too many orders and had to give priority to its regular customers. A Ltds employees went out on strike. A number of A Ltds key workers were absent through illness.

1.8

Which of the following statements is CORRECT? (i) (ii) (iii) A condition is a term which the parties intended to be of fundamental importance. A warranty is a term which the parties did not intend to be of fundamental importance. If an innominate term is broken the innocent party has the option whether or not to terminate the contract.

A B C D

(i) only. (i) and (ii) only. (ii) and (iii) only. (i), (ii) and (iii).

1.9

DE Ltd contracted to deliver a quantity of goods to F Ltd to the value of 5,000. The goods were delivered and DE Ltd submitted an invoice to F Ltd for the amount due which contained a number of new terms.

Which ONE of the following statements is INCORRECT? A B C D The invoice is a contractual document and F Ltd is bound by the terms on the invoice. F Ltd is only bound by the terms if it was given notice of them at or before the time of contract. F Ltd is only bound by the terms if there has been a sufficient course of dealings between DE Ltd and F Ltd so that F Ltd is assumed to know of the terms. If F Ltd is unaware of the terms, it can only be bound by them if it agrees to be so.

FBLW

November 2003

FOR FREE CIMA, ACCA & CAT RESOURCES VISIT: http://kaka-pakistani.blogspot.com

1.10 Which ONE of the following remedies is NOT available for a breach of a contract to provide personal services? A B C D Damages. A decree of specific performance. An injunction. Rescission.

1.11 Which ONE of the following remedies is NOT available for claims of unfair dismissal? A B C D Compensation. Re-instatement. Re-engagement. Specific performance.

1.12 Dan agreed in his contract of employment that he would not work for a competitor of Gee Ltd within the first 12 months after leaving the companys employment. On leaving Gee Ltd, Dan worked for Zed Ltd, a competitor of Gee Ltd. If the restriction in Dans contract with Gee Ltd should be found to be reasonable, which of the following remedies would Gee Ltd be entitled to? (i) (ii) (iii) A B C D (i) only. (i) and (ii) only. (ii) and (iii) only. (i), (ii) and (iii). A decree of specific performance forcing Dan to comply with his contract with Gee Ltd. Damages in respect of any loss caused by Dans breach of contract. An injunction to stop Dan working for Zed Ltd.

1.13 Bernards employment as a part-time store manager with Exe Ltd commenced three years ago. Following a meeting with other employees of Exe Ltd, it was agreed that Bernard be appointed Health and Safety Representative. Bernard agreed that as part of this role he would insist on having monthly meetings with the management of Exe Ltd. Exe Ltd considered this to be excessive and dismissed Bernard giving him one weeks notice. What is the legal position? A B C D Bernard cannot claim unfair dismissal because he does not have the relevant period of continuous employment. Bernard cannot claim unfair dismissal because he is a part-time worker. Bernard can claim unfair dismissal. Bernard can claim wrongful dismissal.

November 2003

FBLW

FOR FREE CIMA, ACCA & CAT RESOURCES VISIT: http://kaka-pakistani.blogspot.com

1.14 Which ONE of the following statements is INCORRECT? A B C D In order to register a company, promoters must include a Memorandum of Association in the documents submitted to the Registrar of Companies. A company cannot be registered with the same number as a company already on the register. A company must include a statement of authorised capital in its Articles of Association. The directors of a newly-registered company cannot also act as directors of another company.

1.15 Which ONE of the following statements is CORRECT? A B C D A company can enter into any transaction as the doctrine of ultra vires has been abolished. The directors may ratify an ultra vires transaction. A third party who has acted in good faith may enforce an ultra vires transaction against the company. The shareholders in a public company may change the objects clause by passing a written resolution.

1.16 Which of the following statements is CORRECT? (i) (ii) (iii) A B C D Purchasing a "shelf company" enables business to commence more quickly. It is generally cheaper to purchase a "shelf company" than to arrange for a solicitor or accountant to register a new company. Incorporating a company by registration enables the companys documents to be drafted to the particular needs of the incorporators.

(i) and (ii) only. (ii) and (iii) only. (i) and (iii) only. (i), (ii) and (iii).

1.17 Which ONE of the following statements is CORRECT? A B C D The shares of all public limited companies are quoted on the Stock Exchange. The company secretary of a public limited company must be qualified. A private limited company must have at least two shareholders. A public limited company cannot trade until it has paid-up share capital of at least 50,000.

FBLW

November 2003

FOR FREE CIMA, ACCA & CAT RESOURCES VISIT: http://kaka-pakistani.blogspot.com

1.18 Which of the following clauses in the Articles of Association of Dee Ltd would NOT be enforceable against the company as a breach of contract? A B C D Shareholders shall be paid dividends in cash. All shareholders are entitled to attend and vote at general meetings of the company. Tom (a shareholder) shall be the companys managing director for life. Shareholders wishing to sell their shares shall offer them to the directors who will purchase them at a fair price as determined by the auditors.

1.19 Bee Ltd has an issued share capital of 1,000 ordinary shares of 1 each. Some of the shareholders would like to pass an elective resolution in order to dispense with the need to re-appoint the auditor annually. What is the minimum number of votes which must be cast in order to pass the resolution? A B C D 500 501 750 1,000

1.20 Which ONE of the following statements is INCORRECT? A B C D A floating charge is a charge over a class of company assets which the company is unable to deal with freely in the ordinary course of business. A floating charge must be registered at Companies House within 21 days of its creation otherwise the charge is void against the other creditors. A floating charge must be registered at the companys registered office, but a failure to do so does not affect the validity of the charge. The proceeds from the sale of the assets comprised in the floating charge must first be used to pay off the preferential creditors.

1.21 Which ONE of the following statements does NOT apply to a private company which intends to provide assistance for the purchase of its own shares? A B C D The shareholders must pass a special resolution. The directors must make a declaration of solvency. The financial assistance must be approved by the court. The company must have authority to provide financial assistance in its Articles of Association.

November 2003

FBLW

FOR FREE CIMA, ACCA & CAT RESOURCES VISIT: http://kaka-pakistani.blogspot.com

1.22 Which ONE of the following statements is INCORRECT? A B C D A private company must have at least two directors. The board usually has the power to appoint a managing director. A board usually reaches its decisions by majority vote. The first directors of the company are those persons who are named as directors on Form 10.

1.23 Which ONE of the following statements is INCORRECT? A B C D Individual directors cannot contract on behalf of the company unless they are authorised by the board. The board is the agent of the company. If the company has appointed a managing director, he or she is able to contract on behalf of the company. The board is appointed to act as the agent of the shareholders.

1.24 Which of the following are NOT owed fiduciary duties by the directors? (i) (ii) (iii) A B C D The company as a whole. Individual shareholders. All the present and future shareholders.

(i) and (ii) only. (ii) only. (i) and (iii) only. (iii) only.

1.25 Which of the following may be liable to contribute to the assets of the company in the event of the company carrying on business at a time when the directors ought to have known that insolvency was inevitable? A B C D The current directors and shareholders of the company. The current and former directors of the company. The current directors of the company. The current directors and former shareholders of the company.

Total = 50 Marks

End of Section A
FBLW 8 November 2003

FOR FREE CIMA, ACCA & CAT RESOURCES VISIT: http://kaka-pakistani.blogspot.com

SECTION B 50 MARKS ANSWER ALL THREE QUESTIONS


Question Two
Purchaser Ltd ("Purchaser") contracted with Builder Ltd ("Builder") for the construction of new luxury business premises at a cost of 3 million. The building was to contain office accommodation and a "leisure suite" including a conference centre, restaurant, fitness centre and indoor swimming pool. Builder departed from the strict specifications of the contract and used sub-standard materials. After using the building for only six months, Purchaser had to move to alternative premises in order that repairs could be carried out. In consequence, Purchaser claimed damages in respect of the following: (i) (ii) The cost of the only alternative office accommodation available for the four months that the building had to be closed for repairs: Total 200,000. Purchaser had contracted with Consultant plc to use the building as the venue for the "International Consultant Conference 2003". Purchaser had to withdraw due to the non-availability of the building: Total amount of profit lost 50,000. Do not write in these columns below
For use Marks by the first available marker For use by the second marker

Required (a) Delete as appropriate* and complete this sentence.


"Purchaser will* /will not* be entitled to recover the 200,000 cost of alternative accommodation from Builder because

. " (Maximum of 30 words) sub-total:

3 4 1

(b) Delete as appropriate* and complete this sentence.


"Purchaser will* /will not* be entitled to recover the loss of profit of 50,000 from Builder if

."

(Maximum of 30 words) sub-total:

3 4

November 2003

FBLW

FOR FREE CIMA, ACCA & CAT RESOURCES VISIT: http://kaka-pakistani.blogspot.com

Question Two continued


Do not write in these columns below
For use Marks by the first available marker For use by the second marker

Required: (c) Delete as appropriate* and complete this sentence.


"Purchaser may* /may not* be liable to Consultant plc for breach of contract as it will be able to argue that the contract was
(maximum 1 word) by the non-availability of its building."

1 2

Sub-total:

(d) Complete this sentence.


"If the contract between Purchaser and Builder had contained a clause excluding liability for faulty workmanship, the clause would have been
(maximum 3 words)

2 1 3

under the 1997." (maximum 3 words)

Act Sub-total:

(e) Complete these sentences.


"If the contract between Purchaser and Builder had contained a "liquidated damages clause" restricting any claim for breach of contract to 100,000, then Purchaser would have been bound by the clause, as long as it was a genuine attempt to

." (Maximum of 10 words) If, however, the clause was found to be a penalty clause, it would be
(maximum 1 word) against Purchaser.

1 sub-total: 3

Total for Question Two = 17 Marks

FBLW

10

November 2003

FOR FREE CIMA, ACCA & CAT RESOURCES VISIT: http://kaka-pakistani.blogspot.com

Question Three
DEF plc is a manufacturer and wholesaler of office furniture. The board decided that the company should extend operations into retailing its products and, in consequence, is proposing to do the following: (i) (ii) To register Retailer Ltd as a wholly-owned subsidiary to own and operate the retailing business. DEF plc to obtain finance for Retailer Ltd by borrowing 3 million from Lender Bank plc, subject to a fixed charge over its factory premises; and by raising 2 million in Do not write in these share capital by issuing a further one million ordinary 1 shares at a columns below price of 2 per share.

Required:
Marks available

For use by the first marker

For use by the second marker

(a) State the four documents that DEF plc will arrange to be sent to the
Registrar of Companies in order to register Retailer Ltd. (i) (ii) (iii) (iv) sub-total: 1 1 1 1 4 1 sub-total: 1 2

(b) State the two characteristics of a fixed charge.


(i) (ii)

(c) Delete as appropriate* and complete these sentences.


"The new shares must first be offered to
(maximum of 4 words). The

1 1 1 1 4

difference between the nominal value of the shares and the price (maximum 1 word) and this must charged is called a be transferred to a
(maximum of 3 words) and may*/may not* be distributed as a

dividend." sub-total:

(d) Delete as appropriate* and complete this sentence.


"If retailing should prove unsuccessful so that Retailer Ltd has to be liquidated, DEF plc will*/will not* be liable for the debts of Retailer Ltd because 1

."

(Maximum of 30 words) sub-total:

3 4

November 2003

11

FBLW

FOR FREE CIMA, ACCA & CAT RESOURCES VISIT: http://kaka-pakistani.blogspot.com

Question Three continued


Do not write in these columns below

Required (e) Delete as appropriate* and complete this sentence.


" DEF plc */Retailer Ltd * will be legally responsible for the actions of employees of Retailer Ltd, so long as they were acting

Marks available

For use by the first marker

For use by the second marker

."

(Maximum of 30 words) sub-total:

2 3

Total for Question Three = 17 Marks

FBLW

12

November 2003

FOR FREE CIMA, ACCA & CAT RESOURCES VISIT: http://kaka-pakistani.blogspot.com

Question Four
The Board of XYZ Ltd has resolved to carry out the following transactions: (i) (ii) (iii) To appoint Imran as a director and Kim as Company Secretary. To alter the companys constitution to facilitate the purchase of its own shares. To purchase the business of Zed Ltd and provide consideration in the form of ordinary 1 shares. Do not write in these columns below
For use by the first marker For use by the second marker

Required: (a) Delete as appropriate* and complete this sentence.


"Subject to the companys Articles */Memorandum * of Association, the board may appoint Imran as a director. His appointment must be confirmed by the shareholders by resolution." (maximum 1 word in each space) or

Marks available

1 1+1

(b) Delete as appropriate* and complete these sentences.


"Kim may be appointed Company Secretary by the company, (maximum 1 (maximum 1 word) as this is a
word) Kim must* /need not* be professionally qualified."

1 1 1 1 1+1

(c) Delete as appropriate* and complete these sentences.


"The Articles of Association must be altered by the board* / shareholders* by or resolution
(maximum 1 word in each space). If the board ignores the rights of

shareholders as set down in the Articles, the shareholders may sue the
(maximum 1 word) for breach of contract."

1 1 1 1 1 1 ." 1

(d) Delete as appropriate* and complete these sentences.


"The business of Zed Ltd must* /need not* be valued by XYZ Ltds auditor. If the valuation were to be carried out negligently, the auditor could be liable to the directors* / company* for ." (maximum of 3 words).

(e) Complete this sentence.


"Although a companys auditors do not have a
(maximum of 1 word) relationship with shareholders, the latter may be

able to sue the auditors in the


(maximum of 3 words) if they can establish that the auditors owed them (maximum of 4 words).

Total for Question Four = 16 Marks

End of Paper
November 2003 13 FBLW

FOR FREE CIMA, ACCA & CAT RESOURCES VISIT: http://kaka-pakistani.blogspot.com

Vous aimerez peut-être aussi